Path - Microbiology Flashcards

1
Q

Match each numbered class of antibiotic with its lettered mechanism of action/ description

  1. Beta lactams
  2. Oxazolidinones
  3. Glycopeptides
  4. Aminoglycosides & tetracyclines
  5. Macrolides & chloramphenicol
  6. Fluoroquinolones
  7. Nitroimidazoles
  8. Rifamycins
  9. Sulphonamides and diaminopyrimidines

A. Inhibits a subunit of DNA gyrase to disrupt DNA replication
B. Binds to pentapeptides to prevent them from being used to form peptidoglycan
C. Binds to the 50s subunit of bacterial ribosomes
D. Interferes with folic acid metabolism, indirectly affecting DNA replication
E. Binds to the 30s subunit of bacterial ribosomes
F. Inactivates transpeptidases which are important for incorporating peptidoglycan into the bacterial cell wall
G. Binds to the 23s component of the 50s subunit of bacterial ribosomes to prevent formation of the initiation complex
H. Produces an active intermediate under anaerobic conditions which causes DNA strand breakage
I. Inhibits protein synthesis by binding to DNA-dependent RNA polymerase thereby inhibiting initiation

A
  1. Beta lactams - F. Inactivates transpeptidases which are important for incorporating peptidoglycan into the bacterial cell wall
  2. Oxazolidinones - G. Binds to the 23s component of the 50s subunit of bacterial ribosomes to prevent formation of the initiation complex
  3. Glycopeptides - B. Binds to pentapeptides to prevent them from being used to form peptidoglycan
  4. Aminoglycosides/ tetracyclines - E. Binds to the 30s subunit of bacterial ribosomes
  5. Macrolides/ chloramphenicol - C. Binds to the 50s subunit of bacterial ribosomes
  6. Fluoroquinolones - A. Inhibits a subunit of DNA gyrase to disrupt DNA replication
  7. Nirtoimidazoles - H. Produces an active intermediate under anaerobic conditions which causes DNA strand breakage
  8. Rifamycins - I. Inhibits protein synthesis by binding to DNA-dependent RNA polymerase thereby inhibiting initiation
  9. Sulphonamides and diaminopyrimidines - D. Interferes with folic acid metabolism, indirectly affecting DNA replication
How well did you know this?
1
Not at all
2
3
4
5
Perfectly
2
Q

A patient being treated for a separate infection develops diarrhoea, fever, and nausea. They are diagnosed with moderate pseudomembranous colitis.

Which would be the most appropriate antibiotic to give?

A. Ceftriaxone
B. Metronidazole
C. Gentamicin
D. Vancomycin
E. Cefotaxime
A

B. Metronidazole

Metronidazole is used to treat mild-moderate C. difficile colitis, with vancomycin and fidaxomicin being reserved for severe disease. This is partially to avoid resistance to vancomycin developing, and partially because it is nephrotoxic and requires careful monitoring of the patient’s renal function. Cephalosporins should not be used in this case as they are associated with causing C. difficile colitis. Most antibiotics have some association with C. diff diarrhoea, especially: clindamycin, cephalosporins, fluoroquinolones, and broad spectrum penicillins (the broader the spectrum, the more likely diarrhoea is).

How well did you know this?
1
Not at all
2
3
4
5
Perfectly
3
Q

A known cystic fibrosis patient presents to hospital with purulent cough, fever, and fatigue. Pseudomonas infection is diagnosed.

What would be the most appropriate antibiotic to give?

A. Tetracycline
B. Cefotaxime
C. Ketoconazole
D. Vancomycin
E. Gentamicin
A

E. Gentamicin

Though they are ototoxic and nephrotoxic, and so levels must be monitored, gentamicin and tobramycin (both aminoglycosides) are particularly effective against P. aeruginosa infection.

Pseudomonas can be treated with aminoglycosides (e.g. amikcacin, gentamicin, tobramycin) or with broad-action beta lactams (tazocin, meropenem, ceftazidime) or fluoroquinolones (ciprofloxacin).

How well did you know this?
1
Not at all
2
3
4
5
Perfectly
4
Q

Match the numbered example to the lettered mechanism of resistance (lettered options may be used more than once)

  1. ESBL E. coli resistance to ceftriaxone
  2. Resistance to macrolides
  3. Resistance to trimethoprim and sulphonamides
  4. MRSA resistance to flucloxacillin
  5. Resistance to Rifampicin
A. Impaired uptake of the antibiotic
B. Alteration of the target
C. Enzymatic inactivation of the antibiotic
D. Enhanced antibiotic efflux
E. Bypass of antibiotic-sensitive step
A
  1. ESBL E. coli resistance to ceftriaxone - C. Enzymatic inactivation of the antibiotic
  2. Resistance to macrolides - B. Alteration of the target
  3. Resistance to trimethoprim and sulphonamides - E. Bypass of antibiotic-sensitive step
  4. MRSA resistance to flucloxacillin - B. Alteration of the target
  5. Resistance to Rifampicin - B. Alteration of the target

ESBL stands for extended-spectrum beta lactamase, and organisms which produce it can inactivate a wide range of antibiotics including third generation cephalosporins.

Flucloxacillin-resistant strains (e.g. MRSA) have altered transpeptidase enzymes (penicillin-binding proteins) which prevent beta-lactams from binding. However Flucloxacillin is stable against beta lactamases.

NB: If in doubt, say altered target

How well did you know this?
1
Not at all
2
3
4
5
Perfectly
5
Q

A woman is brought to hospital with suspected meningitis. Blood is taken and an LP is performed before commencement of empirical antibiotics. Upon analysis the CSF shows clear fluid with raised lymphocyte and protein counts, but normal glucose.

What is the most likely causative organism?

A. Haemophilus influenzae
B. Streptococcus pneumoniae
C. Mycoplasma tuberculosis
D. Enterovirus
E. Herpes simplex
A

D. Enterovirus

This is a case of aseptic meningitis: viral meningitis. It is so named because the causative organism does not cause sepsis, and is the most common form of CNS infection. This CSF result usually indicates viral meningitis, for which non-polio enteroviruses (i.e. echoviruses and coxsackie viruses) are responsible for 80-90% of cases. This condition most frequently occurs in children under 1 year old, and presents with photophobia, neck stiffness, and photophobia, and a non-specific rash may also be seen. However the course of the disease is self-limiting and resolves in 1-2 weeks.

It is worth looking up and memorising the CSF findings in each different type of meningitis.

How well did you know this?
1
Not at all
2
3
4
5
Perfectly
6
Q

A 65 year-old man, who is a big fan of unpasteurised cheese, presents with headache and neck stiffness. Blood cultures produce a Gram-positive rod.

What is the most likely causative organism?

A. Listeria monocytogenes
B. Cryptococcus neoformans
C. Haemophilus influenzae
D. Streptococcus pneumoniae
E. Neisseria meningitidis
F. Escherichia coli
G. Mycobacterium tuberculosis
A

A. Listeria monocytogenes

Listeria monocytogenes is one of the three most common causes of meningitis in immunocompromised (including elderly and very young) patients along with group B Streptococcus and Escherichia coli. The relatively advanced age (>50) of the patient, his penchant for unpasteurised cheese, and the presence of a Gram-positive rod imply the cause is Listeria monocytogenes.

How well did you know this?
1
Not at all
2
3
4
5
Perfectly
7
Q

A 33 year-old man presents with headache and neck stiffness. Treating a CSF sample with the Indian ink stain reveals the causative organism.

What is the most likely causative organism?

A. Listeria monocytogenes
B. Cryptococcus neoformans
C. Haemophilus influenzae
D. Streptococcus pneumoniae
E. Neisseria meningitidis
F. Escherichia coli
G. Mycobacterium tuberculosis
A

B. Cryptococcus neoformans

The India ink stain is useful for staining the polysaccharide capsule surrounding Cryptococcus neoformans (which is a yeast). Cryptococcus is a less common cause of meningitis and should not occur unless the patient is in some way immunocompromised. Accordingly, this patient should be offered an HIV test.

How well did you know this?
1
Not at all
2
3
4
5
Perfectly
8
Q

Which option describes the best treatment for meningitis to be given as soon as blood cultures and CSF have been taken if the causative organism is unknown?

A. Meropenem 6g/d or Ceftazidime 6g/d
B. Aciclovir 10mg/kg I.V. tds, Ceftriaxone 2g I.V. bd with Amoxicillin 2g I.V. 4-hourly if >50 or immunocompromised
C. Ceftriaxone 4g/d or Chloramphenicol 75-100mg/kg/d
D. Ampicillin 12g/d plus Gentamicin or Benzylpenicillin 18-24 mg/d
E. Ceftriaxone 2g I.V. bd with Amoxicillin 2g I.V. 4-hourly if >50 or immunocompromised

A

E. Ceftriaxone 2g I.V. bd with Amoxicillin 2g I.V. 4-hourly if >50 or immunocompromised

The management protocol for meningitis is to immediately take blood cultures and perform an LP. An LP must not be performed if there are signs of raised intra-cranial pressure (decreased GCS, papilloedema on fundoscopy, seizures, or focal neruological deficits). As soon as this has been done, empirical treatment with Ceftriaxone 2g I.V.bd with Amoxicillin 2g I.V. 4-hourly if >50 or immunocompromised is begun. Once culture results are available and the organism is known, more targeted antibiotic regimes can be started. Aciclovir is added to this starter regime if there is suspicion of viral meningio-encephalitis.

NB: Amoxicillin is added in very young or elderly or immunocompromised patients as they are vulnerable to listeria monocytogenes, which will not be killed by Ceftriaxone.

NB: Ceftriaxone is not used in neonates because it displaces bilirubin from albumin which may cause biliary sludging - cefotaxime is used instead

How well did you know this?
1
Not at all
2
3
4
5
Perfectly
9
Q

A 19 year-old man returns from holiday in Spain. Four weeks later, he develops a hot, swollen, painful, red knee joint, with an effusion. The knee is tapped, and 20ml of cloudy yellow fluid is withdrawn. Microbiology reveals Gram-negative intracellular diplococci.

What is the most likely causative organism?

A. Neisseria meningitidis
B. Haemophilus influenzae
C. Streptococcus viridans
D. Escherichia coli
E. Streptococcus pneumoniae
F. Neisseira gonorrhoeae
G. Staphylococcus Aureus
A

F. Neisseria gonorrhoeae

STIs can lead to septic arthritis if untreated, and this should be suspected particularly in young patients or patients with a suspicious sexual/ travel history.

How well did you know this?
1
Not at all
2
3
4
5
Perfectly
10
Q

A previously well 19 year-old student arrives in casualty septic, pyrexial, and confused, with a temperature of 39 degrees. He has a stiff neck, and an LP is performed

Which of the following is the LP most likely to show?

A. Gram positive bacilli
B. Gram negative bacilli
C. Gram positive coccobacilli
D. Gram negative cocci in large clusters
E. Gram positive cocci in chains
A

E. Gram positive cocci in chains

Gram positive cocci in chains implies the pathogen is Streptococcus pneumoniae - one of the three most commonly responsible pathogens in meningitis of immunocompetent patients along with Neisseria meningitidis and Haemophilus influenzae. The three most common organisms causing meningitis in immunocompromised patients are group B Strep (Streptococcus agalactiae), Listeria monocytogenes, and Escherichia coli.

‘A’ describes Listeria and ‘B’ describes E. coli. ‘C’ incorrectly describes H. influenzae, as it is actually a Gram negative coccobacillus. ‘D’ incorrectly describes N. meningitidis, which is a Gram negative diplococcus.

How well did you know this?
1
Not at all
2
3
4
5
Perfectly
11
Q

A 6 year-old boy arrives in casualty septic, pyrexial, and confused with a temperature of 39 degrees. He has a stiff neck, and a lumber puncture reveals Gram-negative coccobacilli.

What is the most likely causative organism?

A. Neisserisa meningitidis
B. Haemophilus influenzae
C. Streptococcus viridans
D. Escherichia coli
E. Streptococcus pneumoniae
F. Neisseira gonorrhoeae
G. Staphylococcus Aureus
A

B. Haemophilus influenzae

Haemophilus influenzae is a coccobacillus, and so is halfway between a rod and a coccus. Haemophilus influenzae is one of the three main causes of meningitis in immunocompetent individuals (the other two being Streptococcus pneumoniae and Neisseria meningitidis). The main three causes of meningitis which only occur in immunocompromised patients are E. coli, Group B Streptococci (S. agalactiae), and Listeria. These organisms most commonly cause meningitis in neonates.

How well did you know this?
1
Not at all
2
3
4
5
Perfectly
12
Q

A 22 year-old has a mild fever for several months, and no cause can be found. After 2 months, blood cultures come back positive for Gram-positive cocci.

What is the most likely causative organism?

A. Neisserisa meningitidis
B. Haemophilus influenzae
C. Streptococcus viridans
D. Escherichia coli
E. Streptococcus pneumoniae
F. Neisseira gonorrhoeae
G. Staphylococcus Aureus
A

C. Streptococcus viridans

Streptococcus viridans is a slow-growing organism which can cause low-level systemic infection. It normally resides in the mouth, hence dental surgery may allow it to enter the bloodstream. Streptococcus viridans causes subacute bacterial endocarditis as bacteria in the bloodstream adhere to heart valves (usually the mitral, but the tricuspid in IVDU).

Subacute bacterial endocarditis requires prior damage of the heart valves in order to take hold. The extracellular matrix of the valves is accordingly damaged by the body’s inflammatory response (hence subacute bacterial endocarditis is classified as a type III hypersensitivity reaction). Subacute bacterial endocarditis usually affects the mitral valve because it is constantly subjected to the high pressures of the left heart, and so is slightly damaged even in healthy people, which makes it easier for bacteria to infect.

NB: This is not to be confused with rheumatic fever: a type II hypersensitivity reaction occurring after Streptococcus pyogenes infection.

How well did you know this?
1
Not at all
2
3
4
5
Perfectly
13
Q

A 35 year-old patient presents with clustered red pustules on his skin, fever, abdominal pain, and blood in the stool. A routine HIV test is positive. During the history the patient reveals they bought a kitten several months ago.

What is the most likely diagnosis?

A. Toxoplasmosis
B. Viral haemorrhagic fever
C. Q fever
D. Bacilliary angiomatosis
E. Brucellosis
A

D. Bacilliary angiomatosis

This infection has been caused by Bartonella henselae, which is a slightly curved Gram-negative rod. Bartonella is found on cat’s paws, particularly in kittens (which also scratch more than adult cats as they cannot retract their claws). Bartonella usually only causes Cat-scratch disease, which features a macule (later pustule as the centre ulcerates) at the site of inoculation, along with regional adenopathy and systemic flu-like symptoms. In some, rarer cases, Cat-scratch disease can cause more severe manifestations e.g. pneumonia, arthritis, hepatitis.

In immunocomprimised patients, Bartonella can cause bacilliary angiomatosis - the formation of vascular lesions within multiple organ systems leading to bleeding. The skin is most often involved, but almost any organ system can be affected (hence the GI bleeding in this patient.

How well did you know this?
1
Not at all
2
3
4
5
Perfectly
14
Q

Why should vancomycin not be given orally apart from in case of pseudomembranous colitis?

A. It is enterotoxic
B. It is not absorbed from the gut
C. It undergoes extensive first-pass metabolism
D. It has been known to rarely cause osteonecrosis of the jaw
E. The oral form is too expensive

A

B. It is not absorbed from the gut

Vancomycin is always given I.V. because it cannot be absorbed form inside the gut. The exception to this is when treating pseudomembranous colitis, as in that case it is ideal to not absorb the vancomycin.

How well did you know this?
1
Not at all
2
3
4
5
Perfectly
15
Q

A 30 year-old man presents with jaundice and haemoptysis. He has felt ‘run-down’ since returning from a canoeing trip to the USA a little over a month ago. On questioning he admits to having muscle aches (especially in his calves), and headaches. On examination there is hepatomegaly and conjunctival suffusion. He reveals that his trip was to a remote region with a lot of rodents, and that there was heavy rainfall during his stay.

What is the most likely causative organism?

A. Rabies lyssavirus
B. Bacillus anthrax
C. Brucella melitensis
D. Bartonella henselae
E. Leptospira interrogans
F. Coxiella burnetii
G. Borrelia burgdorferi
A

E. Leptospira interrogans

Leptospirosis (also known as Weil’s disease) usually presents with sudden onset headache, fever, chills, abdominal pain, muscle pain in the lower limbs (particularly the calf muscle), and conjunctival suffusion (resembles conjunctivitis, but without inflammatory exudates - virtually pathognomic of leptospirosis). The gold standard for diagnosing L. interrogans infection is microscopic agglutination testing.

Jaundice, severe haemoptysis, cardiac arrhythmia, acute renal failure, a maculopapular rash, and rarely mental state changes may all occur in advanced disease. Leptopspirosis consists of two phases: the initial acute/ septic phase features the flu-like symptoms, and the immune phase afterwards which features the more severe and rarer symptoms.

Risk factors include travel to tropical/ endemic regions, travel/ residence in flooding regions especially after heavy rainfall, water sports, and exposure to infected animal urine (particularly rats). Leptospirosis is diagnosed using the Faine’s criteria adopted by the WHO which accounts for clinical, epidemiological, and laboratory findings. Leptospira interrogans can inhabit freshwater for long period of time.

How well did you know this?
1
Not at all
2
3
4
5
Perfectly
16
Q

A 25 year-old man presents to his GP with lethargy for a month accompanied by headaches and fever. On examination, he had a temperature of 39 degrees and splenomegaly. He has recently travelled to Italy. Small Gram-negative coccobacilli were seen on culture with Castaneda’s medium.

What is the most likely causative organism?

A. Rabies lyssavirus
B. Bacillus anthrax
C. Brucella melitensis
D. Bartonella henselae
E. Leptospira interrogans
F. Coxiella burnetii
G. Borrelia burgdorferi
A

C. Brucella melitensis

Brucella is a Gram negative intracellular coccobacillus which causes a zoonotic infection. Brucellosis typically causes undulating fevers, headache, arthralgia, night sweats, and anorexia in its initial stages. Later the infection may become more serious and lead to neural infection, endocarditis, liver abscesses, spondylitis, and epididymo-orchitis.

How well did you know this?
1
Not at all
2
3
4
5
Perfectly
17
Q

A 22 year-old student presents to her GP upon return from a biology field trip with a lesion on her leg which is 5cm in diameter and flat, with a red edge and a dim centre. She also mentions feeling tired and suffering form headaches. On examination, the GP noted a fever of 38 degrees and an irregular heart rate.

What is the most likely causative organism?

A. Rabies lyssavirus
B. Bacillus anthrax
C. Brucella melitensis
D. Bartonella henselae
E. Leptospira interrogans
F. Coxiella burnetii
G. Borrelia burgdorferi
A

G. Borrelia burgdorferi

This is a history of Lyme disease. The best known feature of Lyme disease is an expanding rash known as erythema migrans (seen in ~80% of patients); it has a target/ bullseye appearance and is caused by infection of the skin by Borrelia. The rash appears within the first couple of weeks after a tick-bite, and is accompanied by generic flu-like symptoms.

Once disseminated, Borrelia can cause neurological compliations (facial palsy, radiculopathy, meningitis, encephalitis, peripheral neuropathy), cardiological complications (myocarditis leading to abnormal rhythms - palpitations), and arthritis.

How well did you know this?
1
Not at all
2
3
4
5
Perfectly
18
Q

A tanner on holiday from India presents to hospital with an ulcerating papule on his hand. On inspection of the ulcer, the centre was black and necrotic. Gram-positive rods grew on blood agar culture and responded to treatment with large doses of penicillin.

What is the most likely causative organism?

A. Rabies lyssavirus
B. Bacillus anthrax
C. Brucella melitensis
D. Bartonella henselae
E. Leptospira interrogans
F. Coxiella burnetii
G. Borrelia burgdorferi
A

B. Bacillus anthrax

A tanner works with animal hides, which provide a perfect environment for Bacillus anthrax to proliferate, and so the occupation in this case gives a strong clue.

How well did you know this?
1
Not at all
2
3
4
5
Perfectly
19
Q

A 49 year-old man was admitted from A&E with a 3-day history of worsening right arm pain and a 1-day history of hypersalivation, agitation, and generalised muscle twitching. Vital signs and blood tests were normal but he later became confused. He developed renal failure and died four days later.

What is the most likely causative organism?

A. Rabies lyssavirus
B. Bacillus anthrax
C. Brucella melitensis
D. Bartonella henselae
E. Leptospira interrogans
F. Coxiella burnetii
G. Borrelia burgdorferi
A

A. Rabies lyssavirus

Rabies lyssavirus is transmitted through saliva, and once a person is infected the virus enters their peripheral nervous system and spreads to their CNS. There is an incubation period of 1-3 months, but once the virus spreads to the CNS and the patient become symptomatic, the patient almost invariably dies - only a handful of people have ever survived symptomatic rabies.

Vaccination provides some protection, and prognosis is good if treatment is started before symptoms begin. If unvaccinated, the patient will need three doses of vaccine along with human rabies immunoglobulin, which is 100% effective if given within 14 days If already vaccinated, a person need only receive post-exposure vaccination to be treated.

Symptoms begin with generic flu-like symptoms, then progress to behavioural changes with aggression, paranoia, terror, paralysis, delerium, and eventually coma and death. A treatment known as the Milwaukee protocol was tried which involved placing patients in a chemically induced coma and giving anti-viral treatment, but was not successful and is no longer tried.

Don’t get rabies kids

How well did you know this?
1
Not at all
2
3
4
5
Perfectly
20
Q

A French farmer visiting family in the UK presents with fever and fatigue of two weeks duration. He also admits a dry cough and some abdominal pain. Examination reveals reduced chest expansion and crackles on auscultation, as well as tenderness over the right hypochondriac region and hepatomegaly. The farmer keeps a range of livestock for breeding and has close contact with the animals including assisting in their giving birth.

What is the most likely causative organism?

A. Coxiella burnetii
B. Borrelia burgdorferi
C. Leptospira interrogans
D. Staphylococcus aureus
E. Streptococcus viridans
A

A. Coxiella burnetii

Coxiella burnetti causes Q fever. Q fever is difficult to distinguish from other infections, particularly brucellosis as it shares the same reservoirs (goats, cattle) and has a similar presentation (pneumonia, flu-like illness, hepatitis). Q fever may also result in focal infections in a wide variety of systems. Around 5% of Q fever cases will proceed to a chronic infection, which confers an increased chance of developing endocarditis. Q fever is a significant problem in pregannt women, and has a distinct presentation.

How well did you know this?
1
Not at all
2
3
4
5
Perfectly
21
Q

Why is penicillin less effective on infections with a high bacterial load?

A. The bacteria form dense populations which physically prevents drugs from reaching the centre
B. Horizontal gene transmission of resistance genes is more likely in these populations
C. There is a greater concentration of beta-lactamase produced
D. Greater populations are more likely to include other bacterial species which are not susceptible to penicillin
E. Penicillin is only active whilst bacteria are replicating which occurs less when their numbers are greater

A

E. Penicillin is only active whilst bacteria are replicating which occurs less when their numbers are greater

As bacterial numbers increase, their rate of growth slows, as resources become more scarce. Penicillin only affects bacteria in the growth phase, and so if the bacteria are not growing at a significant rate, penicillin will have no effect.

How well did you know this?
1
Not at all
2
3
4
5
Perfectly
22
Q

Which of the following options most accurately describes a prion disease?

A. A disease with a genetic form inherited in an autosomal recessive fashion
B. A sub-variant of amyloidosis
C. Disease caused by alpha-helical configured proteins
D. A pure protein infectious agent which causes untreatable neurodegeneration
E. A progressive neurodegenerative condition similar to dementia, causing steady decline over many years

A

D. A pure protein infectious agent which causes untreatable neurodegeneration

Prion diseases are caused by the presence of misfolded proteins (prions) in the CNS which cause rapid and untreatable neurodegeneration.

The aetiology of prion disease is not thoroughly understood, as it is incredibly rare (~1 in a million). What is known is that the prion protein (PrP) exists in normal organisms and is not harmful, though its normal function is unknown. PrP can become misfolded to produce a prion. A prion is distinguished by its beta-sheet configuration (PrP normally has an alpha-helical configuration) which makes it incredibly resistant to proteases, which means it accumulates in the CNS, then aggregates to form PrP amyloid plaques which cause neurodegeneration (however prion disease is separate from amyloidosis). Prions are somehow able to misfold other PrP to also become prions, and so behave like infectious agents.

The resilience of prions means that they cannot reliably be removed from surgical instruments, even with extreme heat or radiation. Any instruments used in surgery on a prion-infected patient must be destroyed.

Whilst prion disease is steadily progressive, once symptoms begin it progresses rapidly, causing death in 3 months - 2 years depending on the type. However incubation periods may be extremely extensive, with periods of 45 years being reported amongst Kuru patients in Papua New Guinea.

How well did you know this?
1
Not at all
2
3
4
5
Perfectly
23
Q

What is the most common form of prion disease?

A. Kuru
B. Iatrogenic Creutzfeld-Jacob disease
C. Gerstmann-Straussler-Sheinker syndrome
D. Variant Creutzfeld-Jacob disease
E. Sporadic Creutzfeld-Jacob disease
A

E. Sporadic Creutzfeld-Jacob disease

Sporadic CJD represents ~80% of all prion disease, and affects older people (mean age of onset is 65). SCJD causes rapid dementia with myoclonus, cortical blindness, lower motor neuron signs, and akinetic mutism (dysarthria with akinesia/ ataxia). The cause of sCJD is unknown.

Signs of sCJD include: incresed signal in the basal ganglia on MRI, periodic triphasic complexes on EEG, presence of S100 and 14-3-3 neurodegenerative markers, and spongiform vacuolation and PrP amyloid plaques on biopsy.

How well did you know this?
1
Not at all
2
3
4
5
Perfectly
24
Q

Which statement is false of sporadic Creutzfeld-Jacob disease?

A. Median survival time after diagnosis is <6 months
B. Tonsillar biopsy can be diagnostic
C. EEG usually shows periodic complexes
D. Mean age of onset is 65
E. CSF markers (S100, 14-3-3) of neuronal damage may be elevated

A

B. Tonsillar biopsy is diagnostic

Tonsillar biopsy is of no use in sCJD, but is 100% sensitive and specific in variant CJD.

How well did you know this?
1
Not at all
2
3
4
5
Perfectly
25
Q

Which statement is true of variant Creutzfeld-jacob disease?

A. The disease mainly affects elderly people
B. vCJD is more rapidly progressive than sporadic CJD
C. The initial symptoms are always neurological
D. An MRI typically shows the pulvinar sign
E. EEG is usually abnormal

A

D. An MRI typically shows the pulvinar sign

Variant CJD occurs in younger people (median onset is 26) and is slower progressing than sCJD. It usually presents with psychiatric symptoms (anxiety, paranoia, hallucinations, dysphoria) and neurological symptoms appear later (peripheral neuropathy, ataxia, myoclonus, chorea, dementia).

Signs of vCJD include: a positive tonsillar biopsy (100% sensitive and specific in vCJD), increased signal form the pulvinar nuclei in the posterior thalamus on MRI (pulvinar sign), and florid plaques seen on biopsy.

All bar one patients who have been known to contract vCJD have had the methyinine-methyinine polymorphism at codon 129 of the PRNP gene. It is not known why this association exists.

How well did you know this?
1
Not at all
2
3
4
5
Perfectly
26
Q

Which of the following statements is true?

A. The vast majority of cases of variant CJD have been found to be MM at codon 129 of the PRNP gene
B. Familial prion disease does not cause ataxia
C. In familial prion disease, mutations are usually inherited recessively
D. Familial CJD is more rapidly progressive than sporadic CJD
E. Variant CJD can be indicated by increased signal in the basal ganglia on MRI scans

A

A. The vast majority of cases of variant CJD have been found to be MM at codon 129 of the PRNP gene

All but one case

How well did you know this?
1
Not at all
2
3
4
5
Perfectly
27
Q

A 13 year-old patient presents with throat pain and fever. They deny cough, and examination shows tonsillar exudate. Their modified centor score is 5.

Which treatment should be given?

A. Benzylpenicillin
B. Valaciclovir
C. Co-amoxiclav
D. Vancomycin
E. Aciclovir
A

A. Benzylpenicillin

This is most probably a case of bacterial pharyngitis, as indicated by a high centor score, as a score of 3-4 indicates a 32-56% chance of a Streptococcal cause. As a result, benzylpenicillin should be given for 10 days.

How well did you know this?
1
Not at all
2
3
4
5
Perfectly
28
Q

A 78 year-old man is admitted to A&E with cough, dyspnoea, and confusion. His respiratory rate is 33 and his BP is 85/55. He is given an urgent chest X-ray and diagnosed with pneumonia.

How should this patient be managed?

A. Dishrge home with instructions to family to monitor him and return if symptoms don’t improve, give oral amoxicillin with erythromycin
B. Admit to a respiratory ward and give oral amoxicillin and clarithromycin
C. Consider ICU admission and give I.V. co-amoxiclav with clarithromycin
D. Admit to HDU and begin I.V. vancomycin
E. Admit to ICU, take blood cultures, then immdeiately start I.V. ceftriaxone with amoxicillin

A

C. Consider ICU admission and give I.V. co-amoxiclav with clarithromycin

A CURB-65 score would be used to calculate the predicted mortality, and by extension the severity of this patient’s disease.

Confusion - new onset, defined by an AMTS of 8 or less
Urea - Serum conc. > 7mmol/L
Respiratory rate - >30 breaths per minute
Blood pressure - less than 90 systolic or 60 diastolic
65 - aged 65 or over

This patient has a CURB-65 score of 4 and so should be given I.V. co-amoxiclav with clarithromycin and should be admitted to ITU. A score of 0 or 1 indicates possible home-based care, 2 indicates hospital-based care, and 3 or more indicates ITU admission. It is important to monitor for sepsis and remember the Sepsis 6:

Monitor urine output
Take blood cultures
Serial lactate measurements
Give oxygen
Give empirical antibiotics
Give I.V. saline
How well did you know this?
1
Not at all
2
3
4
5
Perfectly
29
Q

A 65 year-old woman presents with jaundice and abdominal swelling. Examination reveals spider naevi and palmar erythema. She has a mild alcohol history, and denies I.V. drug use. She has three children, the first of which was a difficult birth with significant post-partum haemorrhage when she was 21. She mentions she has recently returned from holiday and that she had seafood from a questionable vendor.

What is the most likely cause of her symptoms?

A. Hepatitis A
B. Hepatitis B
C. Hepatitis C
D. HIV
E. Cytomegalovirus
A

C. Hepatitis C

This is a clinical picture of a woman with acute-on-chronic liver failure (also known as decompensated liver failure) as indicated by ascites (low albumin), spider naevi and palmar erythema (high oestrogens as the liver can no longer metabolise them). The seafood is a red herring (badoom cha) because though that could infect this patient with hepatitis A, it is a self-limiting illness that could not cause this extent of damage.

This woman most likely contracted hepatitis C from contaminated blood products given when she gave birth to her first child, as this was before the advent of hepatitis C screening in the UK (1991).

How well did you know this?
1
Not at all
2
3
4
5
Perfectly
30
Q

Why would amoxicillin be given in addition to ceftriaxone to treat meningitis?

A. Because ceftriaxone is not effective against Listeria
B. Because they have synergistic killing effects on bacteria
C. To give enhanced Gram negative cover
D. Because amoxicillin is effective against organisms which produce ESBLs
E. To cover atypical organisms

A

A. Because ceftriaxone is not effective against Listeria

Ceftriaxone is a third generation cephalosporins with good Gram positive and negative cover, but does not protect against Listeria, an important cause of meningitis in newborns and immunocompromised patients. Cheese may be contaminated with Listeria, and is an important clue in the history.

How well did you know this?
1
Not at all
2
3
4
5
Perfectly
31
Q

Which of the following is an antibiotic effective against a Beta-lactamase-producing organism?

A. Benzylpenicillin
B. Piperacillin
C. Tazobactam
D. Amoxicillin
E. Flucloxacillin
A

E. Flucloxacillin

Benzylpenicillin, piperacillin, and amoxicillin are all antibiotics of the penicillin class which are sensitive to beta-lactamase. Tazobactam is a beta-lactamase inhibitor which is given along with piperacillin, but which is not itself an antibiotic. Flucloxacillin is a member of the penicillin class, but is resistant to beta-lactamase.

How well did you know this?
1
Not at all
2
3
4
5
Perfectly
32
Q

A patient is brought to A&E with the signs of meningism, and an LP is immediately performed. The opening pressure is 35cm.

Which cause of meningitis is associated with an especially high opening pressure on LP?

A. Neisseria meningitidis
B. Coxsackievirus
C. Haemophilus influenza
D. Cryptococcus neoformans
E. Echovirus
A

D. Cryptococcus neoformans

Cryptococcus neoformans is a cause of meningitis usually seen only in HIV+ patients. It is well-known for causing a particularly high opening pressure on LP. Cryptococcus neoformans is a fungus that can be stained for using the India Ink stain.

How well did you know this?
1
Not at all
2
3
4
5
Perfectly
33
Q

Which of the following would not be a contraindication for LP in suspected meningitis?

A. Loss of sensation along the lateral side of the leg
B. Blurring of optic disc margins seen on fundoscopy
C. Unknown INR prior to procedure
D. Seizures
E. The presence of a spreading purpuric rash

A

C. Unknown INR prior to procedure

Unless there is a known clotting issue, this is not a contraindication. Some of the main contraindications to LP are:
GCS reduction below 12
Continuous or uncontrolled seizures
Focal neurology
Papilloedema
Infection at LP site
Cardiac/ respiratory compromise
Thrombocytopenia/ clotting abnormalities
An extensive or spreading purpuric rash (i.e. one which indicates DIC which will derange clotting and make the procedure unsafe)
How well did you know this?
1
Not at all
2
3
4
5
Perfectly
34
Q

A 35 year-old woman presents to A&E with constant high fevers of three days duration. She also reports malaise, headache and constipation. She returned from India a week ago. Her temperature is 39, BP is 120/80, and heart rate is 80. Blood cultures reveal Gram negative rods.

What is the most likely diagnosis?

A. Malaria
B. Dengue fever
C. Typhoid
D. Amoebic liver abscess
E. SARS
A

C. Typhoid

When considering fever in the returning traveller, malaria and typhoid are the most common diagnoses and the most important to remember. Typhoid is caused by Salmonella typhi (though an essentially identical disease - paratyphoid - is caused by Salmonella paratyphi). Typhoid causes constant high fevers, headache, cough, malaise, and constipation (NOT DIARRHOEA), and features an insidious onset.

More distinctive features include rose spots (small red macules caused by bacterial emboli), relative bradycardia (Faget’s sign - the pairing of relative bradycardia with fever as usually a febrile patient would be tachycardic), intestinal haemorrhages resulting from colonisation of Peyer’s patches, and intestinal perforation.

S. typhi has flagella and so is motile. As a result, it can spread throughout the body and so typhoid can be picked up on blood cultures, where it will appear as a Gram negative rod. It can also lie dormant within the gallbladder, which is how the original ‘Typhoid Mary’ had recurrent disease.

How well did you know this?
1
Not at all
2
3
4
5
Perfectly
35
Q

A previously well 32 year-old man presents with breathlessness of 3 weeks’ duration, but no cough. A chest x-ray reveals no abnormality, but on exertional testing his oxygen saturations begin to decrease. Two different types of urinary antigen tests are performed and are negative. He is later diagnosed with pneumonia. Initial treatment is co-amoxiclav with clarithromycin, but he does not improve. A subsequent CT chest shows bilateral ground glass opacities.

What is the most likely cause of his illness?

A. Drug-resistant streptococcus pneumoniae
B. Mycoplasma pneumoniae
C. Pulmonary fibrosis
D. Pneumocystis jiroveci
E. Small cell lung cancer
A

D. Pneumocystis jiroveci

PCP used to stand for Pneumocystis carinii pneumonia, and is still used though the organism has been re-named Pneumocytsis jiroveci. PCP is an AIDS-defining illness and features a non-productive cough along with SOB, fever and generic pneumonia/ infection signs.

The presentation is generally insidious, and may feature a normal or non-specific CXR. A CXR may feature perihilar fine reticular shadowing, but a CT chest may be more helpful and may feature perihilar ground glass opacities particularly in the apical regions of the lungs. Decreased exercise tolerance or reduced oxygen saturations on exercise are classic of PCP. Standard treatment is with Co-trimoxazole (trimethoprim with sulphamexothazole).

The urinary antigen tests mentioned are for pneumococcal and legionella species. each test has very high specificity, but sensitivity of only around 60%, nevertheless negative tests suggest those options are less likely.

NB: PCP increases the risk of a pneumothorax

How well did you know this?
1
Not at all
2
3
4
5
Perfectly
36
Q

A patient presents with fever. They have recently returned from central Africa, where they grew up and to which they frequently travel. The patient recorded their fever at 40 degrees a few days ago, but the fever has now dissipated. The patient develops a widespread rash (parts blanching and parts non-blanching) and muscle ache, and the fever returns a couple of days later. A blood test reveals elevated CRP and a markedly decreased platelet count.

What is the most likely diagnosis?

A. Tuberculosis
B. Malaria
C. Typhoid
D. Familial Mediterranean Fever
E. Dengue fever
A

E. Dengue fever

Dengue fever is caused by a single-stranded RNA flaviviridae virus spread by the Aedes mosquito. Infection with the virus may cause either Dengue fever or Dengue haemorrhagic fever. It is a disease endemic to the tropics, and is classified as a neglected tropical disease, and is an important differential when considering fever in the returning traveller.

Primary infection usually causes Dengue fever (secondary infection may also cause it) which is characterised by fever, severe headache (especially retro-orbital), myalgia, arthralgia, anorexia, abdominal discomfort, and perhaps a maculopapular rash. The fever may be biphasic (saddleback fever) though this is relatively uncommon.

Dengue haemorrhagic fever may feature the symptoms above, but will include some of: petechiae/ echymoses, epistaxis, bleeding from the gums, internal bleeding leading to circulatory compromise.

Reduced platelet count is a defining feature of Dengue haemorrhagic fever, but may also be present in regular Dengue fever. Joint pain is common, and can be used to differentiate Dengue from Chikungunya, an arbovirius which may present similarly, but which features much worse debilitating joint pain.

There are four serotypes of Dengue fever, and infection with one generally confers immunity against that serotype but not the others. The big problem with Dengue fever, is that subsequent infection with another serotype results in a more severe infection. The pathogenesis of this is not fully understood, but it is known that antibodies against one serotype somehow enhance infection by another, and that the immune system plays a significant role in the pathogenesis of haemorrhagic fever. This is relatively rare (affecting only 2-4% of patients with secondary Dengue infection) but highly dangerous.

NB: Dengue fever often is asymptomatic or causes mild, vague symptoms (particularly in young children) in the same way that Polio is usually asymptomatic or mild, but can still be dangerous. The prevalence of Dengue infection means that it still exerts a significant disease burden worldwide.

NB: The rash in this question is a combination of non-blanching and blanching because the patient has developed a petechial rash in addition to the blanching maculopapular rash of Dengue fever.

How well did you know this?
1
Not at all
2
3
4
5
Perfectly
37
Q

A 27 year-old man presents to clinic with 1 week duration: fever, swollen lymph nodes in the neck, malaise, diarrhoea and a widespread maculopapular rash. A routine HIV test is positive, and on questioning the likely exposure to HIV occurred 4 weeks ago.

What is the most likely cause of his symptoms?

A. Influenza infection
B. HIV seroconversion illness
C. Toxoplasmosis infection
D. Cryptosporidium infection
E. Infectious mononucleosis
A

B. HIV seroconversion illness

Seroconversion refers to the time when the immune system begins to create antibodies to a pathogen. In patients with HIV this may be accompanied by a brief and self-resolving flu-like illness. This occurs roughly between 3-12 weeks after infection but varies from person to person.

Influenza and mononucleosis are good differentials for this illness, though the diarrhoea and time frame since exposure are more suggestive of seroconversion illness. Seroconversion illness only occurs in ~50% of patients.

Cryptosporidium and toxoplasmosis are parasitic diseases which should not cause significant disease in immunocompetent hosts. In immunocompromised patients cryptosporidium may cause severe diarrhoeal disease (and can cause mild symptoms in immunocompetent patients), and toxoplasma causes widespread infection but particularly affects the CNS causing abscesses and encephalitis (toxoplasmosis is one of the most common opportunistic infections in HIV/AIDS).

How well did you know this?
1
Not at all
2
3
4
5
Perfectly
38
Q

A 5 year-old boy is brought to the GP by his mother with coryzal symptoms, plus conjunctivitis, cough, and fever. The GP prescribes amoxicillin and sends the boy home. The mother brings the boy back two days later as he has developed a maculopapular rash which began at his hairline, and has now spread down his face onto his trunk. On inspection of the mouth, there are white spots on the buccal mucosa.

What is the most likely diagnosis?

A. Cytomegalovirus infection
B. Influenza plus drug reaction
C. Behcet's syndrome
D. Henoch-Schonlein purpura
E. Measles
A

E. Measles

Measles is an extremely infectious viral disease. It is covered by the MMR vaccine which confers a 93% or 97% rate of protection depending on whether one or two doses are given (according to CDC). The MMR vaccine is very effective, but confers a lower rate of protection against mumps (78-88%).

The classic presentation of mumps is a high fever with the three C’s: conjunctivitis, coryza (upper respiratory symptoms - rhinitis essentially), and cough. This initial phase lasts around three days and begins after a 10-14 day incubation period. During the initial phase, Koplik spots may appear - white spots on the buccal mucosa. After the initial phase the classic measles maculopapular rash appears, starting around the hairline and moving caudally.

How well did you know this?
1
Not at all
2
3
4
5
Perfectly
39
Q

A 46 year-old man presents to A&E with headache. On examination he is cachectic and there are several purple plaques on his skin, and white plaques on the side of his tongue. An HIV test is positive, his viral load is 200,000, and his CD4 count is 9. He is immediately started on HAART, but no action is taken over the headache. 2 weeks later he returns to A&E with a considerably worse headache accompanied by vomiting, neck stiffness, and reduced consciousness.

What is the most likely cause of the symptoms?

A. Subdural bleed
B. Encephalitis
C. TB
D. Immune reconstitution
E. Toxoplasmosis brain abscess
A

D. Immune reconstitution

Immune reconstitution inflammatory syndrome (IRIS) occurs when a patient with a low CD4 count has opportunistic infection, and is then started on anti-retroviral therapy. As the immune system reconstitutes it may cause either an unmasking or a paradoxical IRIS.

In unmasking IRIS, the immune system creates an inflammatory response against a previously unknown opportunistic infection, causing the normal symptoms of the disease. In paradoxical IRIS, there is an inflammatory response causing relapse of symptoms of a prior infection that has already been treated (blood cultures are often sterile).

The purple skin plaques mentioned are Kaposi’s sarcomas - a cancer caused by HHV8 which only appears in immunocompromised patients - and hairy leukoplakia (white plaque on the side of the tongue which cannot be wiped away) which also indicates an immunocompromised state.

How well did you know this?
1
Not at all
2
3
4
5
Perfectly
40
Q

Which is the following does not cause a ring-enhancing lesion on a CT scan of the head?

A. Encephalitis
B. Toxoplasmosis abscess
C. Tuberculoma
D. Primary CNS lymphoma
E. Tumour
A

A. Encephalitis

The acronym for causes of a ring-enhancing lesion on a head CT scan is MAGICAL DR:

M - metastasis
A - abscess (e.g. toxoplasmosis)
G - glioblastoma
I - infarct (subacute phase), inflammation (tuberculoma, neurocystericosis - tapeworm infection of CNS)
C - contusion
A - AIDS-related CNS disease
L - lymphoma (ring-enhancing appearance more common in immunocompromised)

D - Demyleinating disease (classically produces an incomplete ring)
R - radiation necrosis, resolving haematoma

How well did you know this?
1
Not at all
2
3
4
5
Perfectly
41
Q

At what CD4 count are HIV patients generally considered to be at high risk from opportunistic infections?

A. 50
B. 100
C. 200
D. 350
E. 500
A

C. 200

CD4 count is used to monitor the competence of HIV patients’ immune systems, and the lower it drops, the more opportunistic infections may take hold and they more severe these infections will be.

Viral load correlates inversely with CD4 count. HIV infection is comprised of three phases: eclipse, acute, and chronic. During the eclipse phase, the virus spreads and begins to infect cells, establishing a viral reservoir. In the acute phase, the immune system begins to produce antibodies to HIV which may cause a brief seroconversion illness (generic flu-like symptoms). Viral load rapidly peaks in the acute phase, and this is the first time HIV can be detected on a blood test. Viral load rapidly decreases to a point known as the ‘set-point’ which is important in prognosis, and from there chronic infection ensues with steady CD4 loss and progression to AIDS.

This is a link to an excellent Nature review article on HIV with some very good diagrams:
https://www.nature.com/articles/nrdp201535#Sec12

How well did you know this?
1
Not at all
2
3
4
5
Perfectly
42
Q

Via which cell surface molecules does HIV gain entrance to cells?

A. CXCR4 and CCR5 only
B. CD8
C. HLA class I
D. HLA class II
E. CD4 and co-receptors
A

E. CD4 and co-receptors

The primary receptor for HIV is CD4, but co-receptors are also required for entry, most often the chemokine receptors CXCR4 and CCR5. Although HLA molecules are not important for the entry of HIV into host cells, they are important in mediating immunity, as certain HLA I (and to a lesser extent HLA II) subtypes are enriched in the naturally immune population (<1% infected people).

How well did you know this?
1
Not at all
2
3
4
5
Perfectly
43
Q

Which of the following is not a characteristic of beta lactams?

A. They generally won’t cross an intact BBB
B. They are secreted through bile
C. They have a relatively short-half-life
D. They are relatively non-toxic
E. They are cross-allergenic (e.g. penicillin allergy gives a 10% chance of allergy to carbapenems or cephalosporins)

A

B. They are secreted through bile

Beta lactams are renally excreted, and so an altered dose should be considered in patients with renal impairment.

How well did you know this?
1
Not at all
2
3
4
5
Perfectly
44
Q

Which of these antibiotics is most active against Gram negative organisms?

A. Chloramphenicol
B. Clarithromycin
C. Benzylpenicillin
D. Vancomycin
E. Linezolid
A

A. Chloramphenicol

Macrolides (clarithromycin), glycopeptides (vancomycin, teicoplanin), and oxazolidinones (Linezolid) are not generally active against Gram negative organisms. Whilst more advanced penicillins have increasing Gram negative coverage, benzylpenicillin is very narrow-spectrum and is not useful against Gram negative organisms.

How well did you know this?
1
Not at all
2
3
4
5
Perfectly
45
Q

Rank the following in terms of risk of opportunistic viral infections:

A. Solid organ transplant recipients
B. Advanced HIV patients
C. Patients on long-term steroids
D. Allogenic stem cell transplant recipients
E. Cytotoxic chemotherapy patients
A
1 - D. Allogenic stem cell transplant recipients
2 - B. Advanced HIV patients
3 - A. Solid organ transplant recipients
4 - E. Cytotoxic chemotherapy patients
5 - C. Patients on long-term steroids

Although all of these patients will be more vulnerable to opportunistic viral infections, those who have received allogenic bone marrow transplants are most heavily immunosuppressed (although long-term, these patients’ immunosuppression can be tapered off more than in solid organ recipients).

How well did you know this?
1
Not at all
2
3
4
5
Perfectly
46
Q

Which of the following presentations is unlikely to be caused by Varicella-Zoster virus infection of an immunocompromised patient?

A. Shortness of breath with cough and pleuritic chest pain
B. Inflammation of the eye, with rapidly progressing retinal necrosis visible on fundoscopy
C. A patient presenting with right-sided visuospatial neglect and right sided weakness worse in the arm than the leg
D. Widespread purple papular lesions on the skin of a man who has been steadily declining
E. A severely unwell 3 year-old child with a haemorrhagic, necrotic rash

A

D. Widespread purple papular lesions on the skin of a man who has been steadily declining

Varicella-Zoster virus (VZV) usually infect children, giving them the relatively harmless disease of chicken pox. VZV is a member of the herpes virus family (HHV3) and lies dormant in nerve ganglia after initial infection. It can reactivate later in life (particularly in the elderly) to cause shingles - a particularly painful vesicular rash that is confined to a dermatome.

In the immunocompromised VZV is a much more significant problem. Whilst it is generally true that - in a significantly immunocomproimised patient - any pathogen can infect any system, there are certain diseases that are particularly associated with VZV infection. These include pneumonitis (A), acute retinal necrosis (B), cerebral vasculopathy leading to stroke (C), purpura fulminans - acquired protein S deficiency resulting from VZV infection and leading to intravascular thrombosis and haemorrhage into the skin (E).

‘D’ describes Kaposi’s sarcoma - purple/brown papules or plaques that can only appear in the immunocompromised, and are classically associated with AIDS. Kaposi’s sarcoma is caused by HHV8, not VZV.

How well did you know this?
1
Not at all
2
3
4
5
Perfectly
47
Q

Match the lettered opportunistic infection with the numbered virus most likely to have caused it.

A. Post-transplant lymphoproliferative disease
B. Retinitis
C. Progressive multifocal leukoencephalopathy
D. Multi-dermatomal shingles
E. Haemorrhagic cystitis

  1. JC virus
  2. VZV
  3. EBV
  4. BK virus
  5. CMV
A

A. Post-transplant lymphoproliferative disease - 3. EBV
B. Retinitis - 5. CMV
C. Progressive multifocal leukoencephalopathy - 1. JC virus
D. Multi-dermatomal shingles - 2. VZV
E. Haemorrhagic cystitis - 4. BK virus

BK virus is a polyoma virus associated with haemorrhagic cystitis in bone marrow transplant patients, and nephropathy in renal transplant patients

How well did you know this?
1
Not at all
2
3
4
5
Perfectly
48
Q

Please examine the following hepatitis B serology results, which profile is consistent with past (cleared) hepatitis B infection?

A. HBV sAg (+), HBV core Ab (+), HBV sAb (-)
B. HBV sAg (-), HBV core Ab (-), HBV sAb (+)
C. HBV sAg (-), HBV core Ab (-), HBV sAb (-)
D. HBV sAg (-), HBV core Ab (+), HBV sAb (+)

A

D. HBV sAg (-), HBV core Ab (+), HBV sAb (+)

Since the past infection was cleared, there should be no surface antigen remaining. The significance of the presence of core antibodies is that the vaccine for hepatitis B only causes production of surface antibodies, therefore the presence of core antibodies helps differentiate cleared infection from vaccinated.

How well did you know this?
1
Not at all
2
3
4
5
Perfectly
49
Q

Which of the following would indicate endocarditis under the Duke classification clinical criteria?

A. Two separate positive S. viridans blood cultures
B. New tricuspid regurgitation with a fever of 38.5 and Janeway lesions and Osler’s nodes on the hands
C. Two separate positive S. aureus blood cultures with septic pulmonary infarcts and a history of I.V. drug use
D. Roth spots, Janeway lesions, Osler’s nodes, and a previous history of rheumatic heart disease
E. One positive H. influenzae culture in an IVDU with a fever of 39, conjunctival haemorrhage, and Roth spots

A

B. New tricuspid regurgitation with a fever of 38.5 and Janeway lesions and Osler nodes on the hands

Although realistically you would strongly suspect infective endocarditis in every one of these cases, the Duke criteria are very specific in terms of what does and does not confirm it.

The Duke criteria is as follows:
To definitely diagnose infective endocarditis you must have either:
Pathological evidence (histology showing vegetations)
Or Clinical evidence

Clinical evidence is divided into major and minor criteria. To confirm the diagnosis you need 2 major criteria, or 1 major and 3 minor criteria, or 5 minor criteria.

Major criteria:
1) Evidence of microorganisms in the blood
NB: if S. viridans, S. bovis, HACEK group, S. aureus, enterococci, then only two separate cultures are needed. If other organisms are detected, then either two samples >12 hours apart are needed, or all 3 or the majority of 4 samples have to be positive
2) Evidence of endocardial involvement (signs on echocardiogram or new valvular regurgitation)

Minor criteria:

1) Predisposing heart condition or IVDU
2) Fever ≥ 38
3) Vascular events e.g. septic pulmonary emboli, Janeway lesions, intracranial haemorrhage
4) Immune events e.g. Roth spots, Osler’s nodes, glomerulonephritis
5) Cultures or biochemical evidence of infection that does not meet the major criteria
6) Echo findings indicating infective endocarditis but not sufficient to meet the major criteria

Luke Moore says to learn this in his PUO lecture

How well did you know this?
1
Not at all
2
3
4
5
Perfectly
50
Q

Which of the following most decreases the likelihood of a patient having giant cell arteritis?

A. An ESR of 40 (not hugely raised)
B. No scalp tenderness on palpation
C. Age of 37
D. No visual impairment
E. Ferritin of <500
A

C. Age of 37

It is a good rule of thumb that people do not get giant cell arteritis below the age of 50

How well did you know this?
1
Not at all
2
3
4
5
Perfectly
51
Q

Which of the following investigations should be part of a patient’s initial work-up for a pyrexia of unknown origin?

A. Cryoglobulins
B. HIV test
C. CT CAP
D. Anti-dsDNA antibody screen
E. Brucella serology
A

B. HIV test

All patients would ideally get an HIV test on admission to hospital, especially for PUO.

How well did you know this?
1
Not at all
2
3
4
5
Perfectly
52
Q

Which would be the most appropriate test in confirming acute EBV infection 2 weeks after possible exposure in an immunocompetent patient?

A. EBV PCR
B. EBV blood cultures
C. EBV IgG
D. Heterophile antibody test
E. EBV IgM
A

E. EBV IgM

EBV PCR is useful and can be done (and is conceivably a correct answer), but is expensive and ideally not used in this case.

Although the heterophile antibody test is useful and widely used, it may also be positive in CMV infection: EBV IgM is more specific. You can’t generally culture viruses.

53
Q

Match the TB drug with the side effect

A. Rifampicin
B. Isoniazid
C. Pyrazinimide
D. Ethambutol

  1. Non-gout arthralgia
  2. Peripheral neuropathy
  3. Optic neuritis
  4. Enzyme inducer
A

A. Rifampicin - 4. Enzyme inducer
B. Isoniazid - 2. Peripheral neuropathy
C. Pyrazinimide - 1. Non-gout arthralgia
D. Ethambutol - 3. Optic neuritis

Rifampicin also causes red secretions (urine, sweat, tears)
Isoniazid must be given with B6 to combat the peripheral neuropathy
Pyrazinimide can trigger gout, but can also cause nont gout-arthralgia
Ethambutol also causes red-green colour blindness

All four can cause GI disturbance and liver toxicity, though pyrazinimide most commonly causes it, so baseline liver function tests must be carried out.
Find Northwick quiz

54
Q

A 19 year-old student arrives in A&E septic, pyrexial, and confused with a fever of 39. Blood cultures fail to grow any organisms, but cold agglutinins are positive.

What is the most likely responsible organism?

A. TB
B. Enteroviruses
C. Herpes virus
D. Neisseria meningitidis
E. Mycoplasma pneumoniae
A

E. Mycoplasma pneumoniae

Mycoplasma pneumoniae is one of the most common causes of community-acquired pneumonia in children and young adults, and is still a significant cause in adults. It is an atypical organism, i.e. it does not have a cell wall, so beta lactams will not be effective against it. Instead, an antibiotic that targets protein production (e.g. a tetracycline or macrolide) will be needed. Mycoplasma pneumoniae typically presents insidiously, with a persistent dry cough and malaise. It can eventually produce severe pneumonia as it has in this patient.

Cold agglutinins are the big key in mycoplasma infection: it triggers an autoimmune reaction which agglutinates and lyses RBCs.

55
Q

Name the five species of Plasmodium

A
Falciparum
Ovale
Vivax
Malariae
Knowlesi
56
Q

1) Why is a 14 day course of primaquine required in addition to a 3-day course of chloroquine for treatment of non falciparum malaria?
2) Which test is very important to perform before giving primaquine?

A

1) Because the life cycle of Plasmodium vivax and ovale include a hypnozoite stage in the liver. The parasite lies dormant within the liver and are not affected by the initial course of chloroquine. However a two-week course of primaquine can kill the hypnozoites; without it, the patient will experience recurrent malaria.
2) G6PD test, because primaquine can trigger serious haemolytic anaemia in patients who are G6PD deficient

57
Q

Which of the following is the most commonly identified cause of fever in a returning traveller?

A) Typhoid
B) Dengue
C) Pneumonia
D) HIV seroconversion
E) Malaria
A

E) Malaria

The three key differentials of fever in the returning traveller are malaria, dengue fever, and typhoid. Amongst them, malaria is comfortably the most common, and if it is falciparum malaria it can be very dangerous.

58
Q

A Nigerian man presents to hospital with a fever feeling generally unwell after returning from Nigeria visiting family and friends. He is tachycardic, with elevated CRP and bilirubin, and decreased haemoglobin. He is HIV negative, but a rapid malaria test reveals falciparum malaria, which is confirmed with thick and thin blood films. His parasitaemia is 20%.

What is the most effective treatment?

A. Co-artem
B. Malarone
C. I.V. quinine
D. I.V. artesunate
E. Oral quinine
A

D. I.V. artesunate

I.V. artesunate has been proven to reduce mortality by 34.7% compared to quinine (SEAQAMAT trial) and so is the best choice for severe falciparum malaria, as indicated by the high parasitaemia. Definition of severe malaria varies by country and organisation, but 20% is severe by any standards.

Although artesunate is ideal, it is a specialist drug which is not always available, in which case I.V. quinine should be the second drug of choice.

Co-artem and malarone are oral treatments for falciparum malaria. Quinine can be used, but is generally avoided as it has more side-effects.

NB: hypoglycaemia is an important and dangerous complication of malaria which should not be forgotten, though its pathophysiology is unclear.

59
Q

A 48 year-old man presents to hospital with abdominal pain and bloody diarrhoea. A brief history is taken, during which he says he has not left the country in years.

Which organism is most likely to be responsible?

A. Enterotoxigenic Escherichia coli (ETEC)
B. Campylobacter jejuni
C. Salmonella typhi
D. Entamoeba histolytica
E. Salmonella paratyphi
A

B. Campylobacter jejuni

ETEC, C. jejuni, and E. histolytica may all cause diarrhoea, but ETEC causes traveller’s diarrhoea - not bloody diarrhoea, though enterohaemorrhagic E. coli (EHEC - the best known serotype being 0157:H7 which can cause haemolytic-uraemic syndrome) can. E. histolytica is unlikely in this case because it is not endemic to the UK, though it is found in North and South America and Asia.

S. typhi and paratyphi cause typhoid and paratyphoid (clinically they are essentially the same) which features constipation, not diarrhoea.

The organisms most associated with bloody diarrhoea are the CHESS organisms:

Campylobacter
Haemorrhagic E. coli
Entamoeba histolytica
Salmonella enteredis
Shigella
60
Q

A 62 year-old man presents with SOB. His family also report he has seemed confused for the past week. He has a significant smoking history and sats are 91% on room air. Chest exam is normal, but biochemistry reveals a sodium of 124. A CXR reveals bilateral interstitial changes.

Given the most likely diagnosis, what is the most appropriate treatment?

A. Co-amoxiclav
B. Tazocin
C. Ceftazidime
D. Doxycycline
E. Surgery to remove malignancy
A

D. Doxycycline

This is a history of pneumonia caused by L. pneumophilia, an atypical organism. The atypical organisms in pneumonia lack a cell wall, so penicillins (amoxicillin in co-amoxiclav and piperacillin in tazocin) are not effective. An antibiotic is needed which affects protein synthesis: either a tetracycline or a macrolide. Macrolides are first line, but are not an option here, so a tetracycline (doxycycline) is the best option.

Other atytpical organisms include Chlamydia psitticai, Mycoplasma pneumoniae, and Coxiella burnetti. These organisms carry a particularly high risk of extra-pulmonary involvement, and will often be more chronic and insidious than typical pneumonia.

Although malignancy could be a differential here, it is not the most likely diagnosis. It should be remembered that both mesothelioma and small cell lung cancer can be associated with hyponatraemia, however the CXR in this case makes it unlikely.

61
Q

A 74 year old woman presents with SOB and right-sided pleuritic pain. She has a fever of 38.5 and her right lung base is dull to percussion with reduced air entry. She is started on doxycycline and cefuroxime but does not improve over the next few days.

What is the most likely diagnosis?

A. TB
B. Empyema
C. Mesothelioma
D. MRSA pneumonia
E. Aspiration pneumonia
A

B. Empyema

Empyema is a collection of pus in the pleural cavity, and can behave like an abscess in that it won’t necessarily respond to antibiotics alone and will need draining.

The failure to respond to antibiotics could also indicate MRSA, mesothelioma, or TB, but the history doesn’t suggest any of these.

62
Q

What property of aciclovir limits its action to infected cells?

A

It is metabolised to its active form by viral thymidine kinase (and other kinases subsequently) and so won’t become active in non-infected cells. When resistance to aciclovir arises, it is almost always through a mutation in viral thymidine kinase (95% of cases) whereas mtuations in viral DNA polymerase are rare (5%).

63
Q

What is the significance of detecting squamous epithelial cells in a mid-stream urine sample?

A

The presence of squamous epithelial cells in an MSU would imply the sample was incorrectly taken. The initial stream should wash away any epithelial cells along with any bacteria colonising the urethra. These bacteria are not a pathological issue as they are not in the bladder, which is why the sample should be mid-stream, so the only bacteria collected have come from the bladder. The presence of squamous epithelial cells reduces the validity of the sample.

The sample is also unlikely to be valid if the culture shows mixed bacterial growth, as in >95% of cases there is only a single organism responsible for a UTI.

64
Q

Which organism is the most common infectious cause of blindness in the world?

A. Neisseria meningitidis
B. Staphylococcus aureus
C. Haemophilus influenzae
D. Staphylococcus epidermidis
E. Chlamydia trachomatis
A

E. Chlamydia trachomatis

A little known fact is that Chlamydia trachomatis is the most common infectious cause of blindness in the world. Though usually though of as an STD, C. trachomatis can be spread from faeces by the musca sorbens fly, and by hand-eye transmission between people. The infection causes inflammation of the inner eyelid. Repeated infections cause the eyelid to scar and turn inwards, leading to corneal scarring and blindness.

65
Q

B is a 61 year old man who attends A+E with a history of fever and cough. He is a heavy smoker but denies any previous health problems. The on call FY2 Dr R sustains a needlestick injury whilst taking an arterial blood gas. Dr R’s consultant is informed and asks B if he would be willing to have an HIV test. B refuses. Dr R does not want to take post exposure prophylaxis unless it is necessary.

A. Dr R should go back and try to persuade B to consent to the test
B. Performing the HIV test without consent would breach B’s article 8 rights
C. Dr R should ask the lab to do a CD4 count instead
D. B must consent to the test as otherwise it would be a breach of Dr R’s human rights under Article 3
E. The hospital must perform the HIV test otherwise it will contravene Dr R’s right to life under article 2

A

B. Performing the HIV test without consent would breach B’s article 8 rights

The four main articles of the Human Rights Act relevant to healthcare are 2, 3, 5, and 8.These are the right to life, prohibition of torture, right to liberty, and right to a private life. Article 8 is the right which protects a patient’s right to confidentiality and to refuse treatment, and so is the most relevant in this case.

66
Q

List the 4 main causes of massive splenomegaly

A

Certain haematological malignancies (e.g. myelofibrosis, CML, hairy cell leukaemia)
Chronic malaria
Leishmaniasis
Gaucher disease

67
Q

A 5 year-old child undergoes a bone marrow transplant from her identical twin sister in an effort to cure her thalassaemia. Which of the following viruses is of particular concern in her case?

A. HHV8
B. Hepatitis C
C. Adenovirus
D. Rotavirus
E. Enterovirus
A

C. Adenovirus

Adenovirus is considered to be a particularly big problem in bone marrow transplant recipients, especially young ones. It can cause disseminated disease and is a significant cause of mortality (~25% overall for untreated asymptomatic patients, but 50% for pneumonia, and 80% for disseminated disease).

68
Q

Which of the following would suggest infection with hepatitis C rather than hepatitis B?

A. Transmission through contact with infected blood
B. A high chance of curing chronic infection using ribavirin and sofosbuvir
C. The development of cirrhosis and hepatocellular carcinoma after chronic infection
D. A significantly greater risk of chronic infection in young children
E. A gap of 12 weeks between exposure and development of symptoms

A

B. A high chance of curing chronic infection using ribavirin and sofosbuvir

Hepatitis C causes chronic infection more often than hepatitis B, and yet treatment (e.g. with sofosbuvir and ribavirin - B) has a ~90% cure rate. Conversely, though it causes chronic infection in <5% people, hepatitis B is not curable.

Both viruses can be transmitted through contact with infected blood. Hepatitis C can also be transmitted vertically from mother to baby, or through sexual practices (particularly those leading to blood exposure), but these are rarer. Hepatitis B is most commonly spread from mother to baby at birth (perinatal) but may also be spread through bodily fluids (e.g. sexually). Blood transfusions before September 1991 were not screened for hepatitis in the UK, so this should be considered in a person who may be presenting with sequelae of chronic hepatitis.

Both viruses can lead to cirrhosis and liver cancer (C). It is known that children in the first year of life who become infected with hepatitis B have a 80-90% chance of establishing a chronic infection, compared to 30-50% of children < 6, or 5% of adults. Accordingly, ‘D’ is true of hepatitis B, not C.

The incubation periods of both viruses are similar and hugely variable, from a few weeks to 6 months, hence ‘E’ is not useful for discriminating between the viruses.

69
Q

A 25 year-old man presents to A&E with an episode of haemoptysis on a background of SOB and increasing chest pain for the past month. He notes show he recently underwent an allogenic bone marrow transplant to cure his thalassaemia. CRP is elevated, and a CT chest features a positive halo sign.

What is the most likely diagnosis?

A. Pulmonary tuberculosis
B. Pneumocystis pneumonia
C. Streptococcus pneumoniae infection
D. Klebsiella pneumonia
E. Pulmonary aspergillosis
A

E. Pulmonary aspergillosis

Aspergillus is generally not an issue in immunocompetent patients, but this individual has recently received an allogenic BMT and so will be heavily immunosuppressed (though with time the immunosuppression for BMT can be significantly tapered off). The major clue to this being aspergillosis is the presence of the ‘Halo sign’ on CT: a ground glass opacity surrounding a pulmonary nodule caused by haemorrhage. The halo sign is not totally specific, but classically indicates invasive aspergillosis.

70
Q

A 74 year old woman becomes acutely confused and disorientated whilst staying in hospital. She is on the renal ward because of renal calculi which have caused a ureteric obstruction and hydronephrosis. Her CRP is elevated and a urine dipstick reveals elevated nitrites and leukocytes.

Which of the following organisms is most likely to be responsible?

A. Pseudomonas aeruginosa
B. Proteus mirabalis
C. Staphylococcus epidermis
D. Streptococcus pneumoniae
E. Staphylococcus saprophyticus
A

B. Proteus mirabalis

This would be defined as a complicated UTI, because it occurs on a background of obstruction and hydronephrosis. Proteus species are a major cause of UTI in patients with structural abnormalities to their urinary tract, though E. coli is still the most common cause. Klebsiella is also a common cause of UTIs.

71
Q

Based purely upon the incidence of different forms of meningitis, which of the following LP findings is most likely in a patient with meningitis?

A. Cloudy fluid, high neutrophils, low glucose, high protein, Gram+ chain cocci cultured from CSF
B. Cloudy fluid, high neutrophils, low glucose, high protein, Gram- diplococci cultured from CSF
C. Clear fluid, high lymphocytes, normal glucose, slightly raised protein, PCR+ for an enterovirus
D. Clear fluid, high lymphocytes, normal glucose, slightly raised protein, PCR+ for a herpes virus
E. Clear fluid, high monocytes, low glucose, high protein, acid-fast bacilli seen on CSF smear

A

C. Clear fluid, high lymphocytes, normal glucose, slightly raised protein, PCR positive for enterovirus

This is essentially just a question of what is the most common cause of meningitis. Viral meningitis is the most common, and it is most commonly caused by echoviruses or coxsackie viruses - i.e. non-polio enteroviruses.

See the CDC page for more info:
https://www.cdc.gov/meningitis/viral.html

72
Q

A 25 year-old woman presents to her GP with suprapubic pain and a burning sensation on urination. She also report increased frequency and urgency, and a small amount of blood in her urine. A urine sample is cloudy and the urine dip shows leukocytes but no nitrites.

Assuming the urine dip result is valid, what is the most likely causative organism?

A. Enterococcus faecalis
B. Escherichia coli
C. Proteus mirabilis
D. Staphylococcus saprophyticus
E. Staphylococcus epidermis
A

D. Staphylococcus saprophyticus

S. saprophyticus is a Gram positive coagulase negative coccus, and is the second most common cause of UTI in young women after E. coli. The urine dipstick result (if assumed to be a valid sample and read correctly) makes the presence of E. coli less likely because of the absence of nitrites. Nitrites are produced from nitrates in the urine by Gram negative bacteria, therefore if the causative organism is Gram positive, nitrites will be negative but leukocytes will be positive.

73
Q

Match the numbered vector with the lettered pathogen it carries:

  1. Tsetse fly
  2. Sandfly
  3. Aedes mosquito
  4. Anopheles mosquito
  5. Ixodes tick
A. Malaria
B. Dengue fever virus/ Chikungunya fever virus
C. Trypanosoma brucei 
D. Leishmania
E. Borrelia burgdorferi
A
  1. Tsetse fly - C. Trypanosoma brucei
  2. Sandfly - D. Leishmania
  3. Aedes mosquito - B. Dengue fever virus/ Chikungunya fever virus
  4. Anopheles mosquito - A. Malaria
  5. Ixodes tick - E. Borrelia burgdorferi

NB: As a rule the Aedes mosquito bits during the day, and the Anopholes mosquito bites at night

74
Q

A 68 year old man presents to A&E with SOB and cough productive of yellow sputum. He was told to go to hospital by his psychiatrist who noticed him coughing during an appointment for his alcohol addiction issues. He is also a type II diabetic who is poorly compliant, and has a significant smoking history. CRP is elevated, and a CXR shows patchy consolidation suggestive of pneumonia.

What is the most likely responsible organism?

A. Streptococcus pneumoniae
B. Klebsiella pneumoniae
C. Staphylococcus aureus
D. Acinetobacter baumannii
E. Pseudomonas aeruginosa
A

B. Klebsiella pneumoniae

K. pneumoniae is a less common organism in pneumonia cases, but is seen in those with underlying conditions (e.g. T2DM), and especially in alcoholics. Once established, Klebsiella has a high mortality rate and can rapidly prove fatal.

NB: the important nosocomial pathogens are the ESKAPE ones:

Enterococcus faecium
Staphylococcus aureus
Klebsiella pneumoniae
Acinetobacter baumannii
Pseudomonas aeruginosa
Enterobacter species (does not include E. coli)

(the first three are Gram+, while the last three are Gram-)

75
Q

A 38 year old woman presents to her GP with a 4 week history of fever, weight loss, malaise, and fatigue. Physical examination reveals painless red spots on the palms, a heart murmur not previously recorded in the patient’s notes, multiple splinter hemorrhages, and splenomegaly. Blood tests reveal anaemia and cultures show a Gram positive coccus. A TOE shows vegetation on the mitral valve of the heart. The patient has been previously well with no medical contact bar a root canal procedure 3 months ago.

What is the most likely diagnosis and causative organism?

A. Subacute bacterial endocarditis, Streptococcus viridans
B. Acute bacterial endocarditis, Coxiella burnetti
C. Rheumatic fever, Streptococcus pyogenes
D. Acute bacterial endocarditis, Staphylococcus aureus
E. Subacute bacterial endocarditis, Enterococcus faecalis

A

A. Subacute bacterial endocarditis, Streptococcus viridans

Streptococcus viridans is an alpha-haemolytic Gram+ coccus which is usually a harmless commensal of the mouth. However during dental procedures it can enter the blood, and can cause subacute endocarditis, a more insidious form of bacterial endocarditis. As it is more chronic, subacute endocarditis is more likely to cause the classic signs of endocarditis (including nail clubbing). The signs of endocarditis can be remembered using the mnemonic ‘FROM JANE’:

Fever
Roth spots
Osler's nodes
Murmur
Janeway lesions
Anaemia
Nail bed haemorrhage
Emboli (causing stroke, PE, haematuria from renal damage)
76
Q

Which of the following pathogens has the highest R0?

A. Measles
B. SARS
C. Ebola
D. Mumps
E. HIV
A

A. Measles

R0 is the average number of healthy people to which one infected individual will pass on an infection. Ebola has an R0 of only 2, SARS and HIV have an R0 of 4, Mumps has an R0 of 10, and Measles is ludicrously contagious with a R0 of 18.

77
Q

A 25 year old MSM man presents to his GP with a fever and RUQ pain having returned from a year spent travelling a week ago. He was mostly well whist abroad and took recommended prophylactic medication, though he has been experiencing diarrhoea for approximately 4 weeks which has recently become bloody. An abdominal exam reveals tenderness and a mass in the RUQ. A liver USS indicates that the mass is an abscess.

What is the most likely diagnosis?

A. Malaria
B. Typhoid
C. Amoebiasis
D. Shigella infection
E. Camplyobacter infection
A

C. Amoebiasis

As a rule, there are 5 main organisms that cause bloody diarrhoea - the CHESS organisms:
Campylobacter jejuni
Haemolytic E. coli
Entamoeba histolytica
Shigella
Salmonella enterica

Entamoeba histolytica infection is an important cause of fever in the returning traveller, and typically will cause a fever whose origin cannot be traced, until a liver abscess is found.

Entamoeba histolytica initially causes bloody diarrhoea, but once it enters the bloodstream it spreads to the liver and establishes an abscess. This may well cause swinging fevers due to intermittent bactaraemia, and so may be difficult to distinguish from malaria.

Risk factors include institutionalisation, being male, being MSM (men who have sex with men), and travel in endemic areas.

78
Q

A baby is diagnosed with sensorineural hearing loss which is believed to be due to congenital viral infection. The likely exposure to the pathogen occurred when the mother was 23 weeks pregnant.

Which of the following pathogens is most likely to have been responsible?

A. Zika
B. Rubella
C. Toxoplasmosis
D. Cytomegalovirus
E. Herpes simplex
A

D. Cytomegalovirus

Though sensorineural hearing loss is included in the classic triad of Rubella syndrome (along with eye and heart abnormalities e.g. cataracts and PDA), cytomegalovirus is the most common non-genetic cause of sensorineural hearing loss in children. Furthermore, Rubella is far less likely to cause abnormalities if a pregnant woman is infected after 20 weeks, meaning that the abnormality in this question is more likely to be due to cytomegalovirus.

NB: HSV is most likely to be transmitted from mother to baby if the mother is infected within 6 weeks of delivery, as there is not time to produce antibodies. Therefore there is a high risk of transmission during vaginal birth, so a C-section is recommended.

79
Q

A 7 day old neonate born at 38+2 is brought by ambulance into the hospital, after it began to develop a petechial rash. The baby quickly became feverish and irritable, then drowsy and floppy, and now has a tense anterior fontanelle. A junior doctor is sent to take a history from the mother: the baby was born vaginally and there was prolonged rupture of membranes. The mother has not has any significant past viral infections bar the flu once or twice, but the junior doctor discovers a possible contact with a viral illness at 35 weeks gestation.

Which of the following viruses is most likely to have caused the baby’s illness?

A. Measles
B. Herpes simplex
C. Rubella
D. Cytomegalovirus
E. VZV
A

B. Herpes simplex

Whilst herpes simplex may cause intrauterine infection (which leads to fairly standard eye, skin, and brain abnormalities) only 5% of infections are transmitted in utero. 85% are transmitted during birth, and particular risk factors for this are prolonged rupture of membranes and vaginal birth. Type of maternal infection is also a significant risk factor; if the mother has never been infected with herpes before, she has a 57% chance of passing the infection to the neonate.

Herpes simplex is most likely to be transmitted to the neonate if the mother is infected within 6 weeks of her delivery, as this does not allow time for the formation of protective antibodies which reduce transmission. Once infected, 45% of neonates develop localised skin, eye, and mouth (SEM) disease. Around 30% will develop CNS disease (which may also feature SEM involvement), and 25% will develop disseminated disease. Disseminated disease may cause a range of complications including pneumonitis, DIC, hepatitis, and frequently CNS involvement, and even with antiviral treatment is associated with major mortality (~30%) and long term neurological sequelae (~17%).

Whilst VZV can cause significant infection in neonates, 90% of women in developed countries are immune by the time they are pregnant, and neonates are only considered significantly vulnerable if the mother is infected within 7 days before or after birth, as there is no time for transfer of passive immunity via antibodies to the neonate. VZV may cause a congenital syndrome but this is fairly uncommon.

NB: In >20% of cases, the neonate will not display the classic skin lesions associated with adult herpes simplex infection, so their absence does not exclude infection.

80
Q

A 26 year old man develops nausea and vomiting a few hours after eating out at a Chinese buffet restaurant. He is surprised he has developed symptoms, as the staff assured him the rice he ate was thoroughly reheated before serving to make sure there were no bacteria in it.

Which of the following bacteria is most likely to have caused his symptoms?

A. Clostridium botulinim
B. Clostridium perfringens
C. Norovirus
D. Rotavirus
E. Bacillus cereus
A

E. Bacillus cereus

Bacillus cereus is classically associated with rice and produces two toxins: a heat-labile one which causes diarrhoeal illness, and a heat-stable one which causes emetic illness. In this case the bacteria have been killed by the thorough reheating of the rice, but the heat-stable emetic toxin has remained and caused the rapid-onset nausea and vomiting.

81
Q

A 34 year old MSM man presents to his GP with a 4 week history of non-bloody, foul-smelling, pale diarrhoea and weight loss. He has recently returned from trekking across the USA for 3 months. The GP refers him to an infectious disease specialist who suspects giardiasis.

Which of the following appearances on microscopy would support a diagnosis of giardiasis?

A. Parasites visible on a thick blood film using a Giemsa stain
B. A mobile trophozoite with 4 nuclei
C. Pear-shaped trophozoites with 4 flagella and 2 nuclei
D. Oocysts seen in the stool using the modified Kinyoun acid fast stain
E. Thin worms seen in stool alongside an eosinophilia on blood tests

A

C. Pear-shaped trophozoites with 4 flagella and 2 nuclei

Giardiasis is a protozoan infection causing a malabsorptive diarrhoea - hence the pale, foul-smelling stools (steathorrea) and weight loss.

Risk factors include MSM (men who have sex with men), being institutionalised in a mental hospital, and travel to an endemic region.

82
Q

Which of the following maternal infections carries the greatest risk of causing in utero infection of the baby if infected within the first trimester?

A. Parvovirus
B. VZV
C. Cytomegalovirus
D. Herpes simplex
E. Rubella
A

E. Rubella

VZV only 0.5%, peaks at 2% between 13-20 weeks
HSV: vulnerable in 3rd trimester, particularly if infected within 6 weeks of birth, low rates of transmission in utero
Parvovirus ~33% before 20 weeks
Cytomegalovirus, though the most common congenital infection, 30-40% transmission
Rubella carries 20% chance of fetal loss, and 90% surviving pregnancies develop congenital Rubella syndrome

83
Q

Which of the following has the lowest infective dose?

A. Entero-toxic Escherichia coli
B. Shigella dysenteriae
C. Vibrio cholerae
D. Salmonella enteritidis
E. Clostridium difficile
A

B. Shigella dysenteriae

Along with norovirus, shigella is considered to have the lowest infective dose of any enteric pathogen, and so can very easily spread between hosts.

84
Q

Which of the following statements regarding PEP (post-exposure prophylaxis for HIV) is correct?

A. It is only effective if taken within 72 hours of exposure
B. It consists of the same drug regimen as is used to treat HIV
C. An RCT has not yet been conducted but once it has will provide better data on efficacy
D. Efficacy is believed to be extremely high
E. PEP should be taken for 2 weeks

A

D. Efficacy is believed to be extremely high

No RCT for PEP efficacy can be conducted because of the obvious ethical issues, but the CDC released a review of 6 observational studies in 2016 which revealed a failure rate of only 0.5%. Even in the cases where PEP ‘failed’ it is though that other factors were responsible and this was not truly down to failure of the medication.

https://www.aidsmap.com/about-hiv/how-effective-post-exposure-prophylaxis-pep

85
Q

What is the natural reservoir of influenza A viruses?

A

Wild birds e.g. ducks

Influenza A viruses are the most significant from a public health perspective as they tend to cause pandemics. This is because they are present in a wide variety of reservoirs and have a strong presence within those reservoir populations. B and C flu viruses cause seasonal flu but have less of a reservoir so don’t tend to cause pandemics

86
Q

Why does the influenza virus cause respiratory disease?

A. The host cell receptor (sialic acid) the virus binds to is only expressed in the lung
B. The influenza virus can only get into the body through the mouth
C. The influenza virus requires activation by host cell proteases that are only expressed in the respiratory tract
D. The influenza virus envelope can only fuse with membranes of cells that secrete mucus
E. The influenza virus is most frequently transmitted via droplets, so the first system it comes into contact with is the respirator system

A

C. The influenza virus requires activation by host cell proteases that are only expressed in the respiratory tract

Haemagglutinin is a viral protein that is essential for its entry into host cells. Haemagglutinin is produced as a precursor by the virus, which must be cleaved by a host protease to become active. The majority of influenza viruses require a very specific protease to perform this action: human airway tryptase.

However influenza viruses may undergo mutations allowing them to be cleaved by proteases found elsewhere in the host body. The avian H5 and H7 subtypes are well known for this property and can kill bird very quickly. They may be transmitted into humans where they cause severe disseminated disease and viraemia because the virus has the ability to replicate anywhere - they are no longer restricted by their tropism.

High viral load in the blood drives production of inflammatory cytokines and leads to cytokine storm, which in turn triggers systemic inflammatory responses e.g. ARDS. It is for this reason that infection with certain subtypes of avian flu (e.g. H5 and H7) are associated with high mortality.

87
Q

What 3 attributes are essential for a pandemic influenza virus?

A

Novel antigenicity
Ability to replicate within the human respiratory tract
Effective transmission between humans

88
Q

What are the three main factors influencing transmission of the influenza virus from animals to humans

A

A switch in affinity of haemagglutinin from the alpha-2,3 to the alpha-2,6 variant of sialic acid

An increase in virion stability conferred by adaptation to a lower pH environment

An increase in the neuraminidase stalk length

89
Q

Why were the elderly less affected by the 2009 swine flu pandemic then expected?

A

Because the virus bore antigenic similarities to an influenza virus circulating between 1920-1940

90
Q

What type of vaccine is given to vulnerable people over the age of 17 for seasonal influenza

A. A live attenuated virus
B. A purified fraction containing HA and NA of an inactivated virus
C. A purified HA protein expressed in insect cells
D. An immunoglobulin fraction from serum of immune patients
E. An adjuvant vaccine

A

B. A purified fraction containing HA and NA of an inactivated virus

Vulnerable children are given the live attenuated quadrivalent vaccine

91
Q

Which mechanisms are responsible for CD4 depletion in HIV infection?

A

Thymic infection reduces production of new T-cells
Incomplete reverse transcript production stimulates inflammation and cell death
Direct cytopathogenicity of HIV replication
Generalised activation of immune response?

92
Q

Which of the following cells are most vulnerable to HIV infection?

A. CD8+ cells
B. Monocyte
C. Dendritic cells
D. Naive CD4 cells
E. Memory CD4 cells
A

E. Memory CD4 cells

Memory CD4+ cells are especially vulnerable to HIV infection because they highly express CCR5 (a co-receptor for HIV entry into cells). Memory CD4+ cells act as a reservoir for HIV infection.

93
Q

A 58 year old man is brought to hospital having been found collapsed on the street. He is semi-conscious, is coughing up mucous, and seems short of breath. A CXR shows an apical pneumonia which is confirmed to be due to Klebsiella pneumoniae by a swab. Several days later he is failing to respond to treatment with meropenem, having also not responded to co-amoxiclav and ceftriaxone.

Which antibiotic is most likely to be effective?

A. Colistin
B. Cefuroxime
C. Vancomycin
D. Daptomycin
E. Tazocin
A

A. Colistin

Colistin is a last-ditch treatment for multi-drug resistant Gram negative infections

94
Q

VSA:

The Ixodes tick carries the pathogen which causes which zoonosis?

A

Lyme disease

*Taken from 2017 paper

95
Q

VSA:

Which drug should be used to treat severe falciparum malaria?

A

I.V. Artesunate

Malaria Treatment Guide:

Non-falciparum - Chloroquine + Primaquine

Falciparum:
Mild (parasitaemia <2%, reasonably well):
Quinine *OR*
Co-Artem *OR*
Malarone

Severe:
I.V. Artesunate
I.V. Quinine if Artesunate unavailable

*Taken from 2017 paper

96
Q

What is the vector for Trypsanoma brucei rhodesiense?

A. Tsetse fly
B. Sandfly
C. Aedes
D. Anopheles
E. Botfly
A

A. Tsetse fly

*Taken from 2017 paper

97
Q

VSA:

How should a pregnant woman who was exposed to VZV 5 days ago be managed?

A

Give her VZIG

NB: shingles doesn’t count

*Taken from 2017 paper

98
Q

What is the most likely cause of fever in a returning traveller?

A

Malaria

*Taken from 2017 paper

99
Q

EMQ:

Match each patient with the organism most likely to be causing their meningitis

A. Herpes simplex virus
B. S. pneumoniae
C. Group B Streptococci
D. E. coli
E. N. meningitidis
F. L. monocytogenes
G. H. influenzae
H. B. melitensis
  1. 74 year old woman with Gram+ bacilli in CSF
  2. Man with meningitis Gram+ diplococci
  3. Young woman with following CSF results: normal glucose, raised lymphocytes, slightly raised protein
  4. 2 day old baby with gram negative rods in CSF
  5. Young guy with meningitis and Gram- diplococci
A
  1. 74 year old woman with Gram + bacilli in CSF - F. L. monocytogenes
  2. Man with meningitis Gram + diplococci - B. S. pneumoniae
  3. Young woman with following CSF results: normal glucose, raised lymphocytes, slightly raised protein - A. Herpes simplex virus
  4. 2 day old baby with gram negative rods in CSF - D. E. coli
  5. Young guy with meningitis and Gram negative diplococci - E. N. meningitidis
    * Taken from 2017 paper
100
Q

EMQ:

Match each scenario with the most likely causative organism. Each antibiotic option may be used once, multiple times, or not at all.

Bacillus Cereus
Norovirus
Rotavirus
Salmonella
Giardia
Campylobacter
Adenovirus
Typhoid
Escherichia Coli
Clostridium Difficile
  1. A young woman develops diarrhoea whilst travelling abroad through rural India
  2. A man develops foul-smelling diarrhoea and cramping pain 5 days after eating chicken at a barbecue
  3. A man presents with vomiting 10 hours after going out for a meal at a Chinese buffet restaurant with a lot of rice
  4. Woman has had some surgery which required antibiotics, and now has profuse watery diarrhoea
  5. A ward sister gets diarrhoea, and patients on the ward have been ill recently with the same symptoms
  6. An MSM man who has just returned from travelling has been experiencing particularly foul-smelling greasy diarrhoea for the past two weeks
  7. A 3 year old unvaccinated child develops vomiting and diarrhoea
A
  1. A young woman develops diarrhoea whilst travelling abroad through rural India - Escherichia Coli
  2. A man develops foul-smelling diarrhoea and cramping pain 5 days after eating chicken at a barbecue - Campylobacter
  3. A man presents with vomiting 10 hours after going out for a meal at a Chinese buffet restaurant with a lot of rice - Bacillus Cereus
  4. Woman has had some surgery which required antibiotics, and now has profuse watery diarrhoea - Clostridium Difficile
  5. A ward sister gets diarrhoea, and patients on the ward have been ill recently with the same symptoms - Norovirus
  6. An MSM man who has just returned from travelling has been experiencing particularly foul-smelling greasy diarrhoea for the past two weeks - Giardia
  7. A 3 year old unvaccinated child develops vomiting and diarrhoea - Rotavirus

NB: Question 2 is a little unfair because this could very reasonably be Salmonella, but Campylobacter is generally acknowledged to be a slightly more common cause of gastroenteritis

*Modified from 2017 paper

101
Q

EMQ:

Match each scenario with the appropriate antibiotic. Each antibiotic option may be used once, multiple times, or not at all.

Amoxicillin
Coamoxiclav
Tazocin
Vancomycin
Flucloxacillin
Trimethoprim
Phenoxymethylpenicllin/ penicillin V
Ciprofloxacin
  1. What is the treatment of choice for Listeria meningitis?
  2. Which antibiotic would be used for a mild CAP (CURB65 of 0 or 1)
  3. Which antibiotic would be used for a severe HAP
  4. Which antibiotic should be used to treat Strep pharyngitis?
  5. Which antibiotic would you prescribe for a young women with cystitis caused by fully sensitive E-Coli?
  6. Which antibiotic would you use for cellulitis with MRSA?
  7. Which antibiotic would you use to treat someone who has cellulitis with an MSSA?
A
  1. What is the treatment of choice for Listeria meningitis? - Amoxicillin
  2. Which antibiotic would be used for a mild CAP (CURB65 of 0 or 1) - Amoxicillin
  3. Which antibiotic would be used for a severe HAP - Tazocin
  4. Which antibiotic should be used to treat Strep pharyngitis? - Phenoxymethylpenicllin/ penicillin V
  5. Which antibiotic would you prescribe for a young women with cystitis caused by fully sensitive E-Coli?
    - Trimethoprim
  6. Which antibiotic would you use for cellulitis with MRSA? - Vancomycin
  7. Which antibiotic would you use to treat someone who has cellulitis with an MSSA? - Flucloxacillin
    * Modified from 2017 paper
102
Q

VSA:

What is the treatment for a pregnant woman infected with VZV?

A

Aciclovir (oral or I.V. depending on severity)

103
Q

Describe the life cycle of HIV

A
Attachment and fusion
Reverse transcription
Integration
Transcription
Protein synthesis
Assembly and exit
Maturation
104
Q

Describe the effects of HIV on CD4+ T-cells:
Which cells are infected?
What effect does this have?
What are the long-term ramifications even after treatment?

A

Memory CD4+ T-cells are more easily infected than other cells
T-cell depletion allows higher loads of other pathogens causing widespread immune activation
Activated cells are more susceptible to HIV infection so this helps the virus proliferate and spread
There is gradual depletion of memory CD4+ T-cells
Even after initiation of successful HAART, immunological memory has been shown to be depleted

105
Q

The massive rate of mutation in HIV is due to low fidelity copying by which two enzymes?

A

Reverse transcriptase and RNA polymerase

106
Q

What is the basic structure of a HAART regimen?

A

Two nucleoside reverse transcriptase inhibitors (NRTIs) + one of:
A protease inhibitor OR
A non-nucleoside reverse transcriptase inhibitor OR
An integrase inhibitor

Integrase inhibitors prevent integration of the viral DNA into the host genome (post-reverse transcription)
Protease inhibitors block cleaving of HIV proteins which is an essential step in maturing the new virion so it may infect other cells

107
Q

Which of the following best predicts the course of HIV infection?

A. Presence and severity of symptoms during seroconversion
B. Peak viral load in primary infection
C. Viral load after primary infection peak settles
D. Lowest CD4+ count in primary infection
E. Peak CD8+ cell count in primary infection

A

C. Viral load after primary infection peak settles

This is known as the viral burden ‘set point’ and is correlated with clinical outcome, with so-called ‘elite controllers’ having very low viral load set points.

108
Q

Which of the following routine tests in an HIV patient would be carried out using flow cytometry?

A. HIV screening test
B. HIV confirmatory test
C. Plasma viral load quantification
D. CD4+ count
E. Drug resistance analysis
A

D. CD4+ count

The screening test is ELISA
The confirmatory test is a Western blot analysis (electrophoresis)
Plasma viral load is quantified using PCR
Drug resistance analysis is done using either cell culture or viral genome sequencing

NB: All these investigations would be carried out in a new HIV patient, and would be done in the order they are written in this question

109
Q

What is the average transmission mother to child inc. breast feeding in untreated population?

A

Around 1/3

110
Q

What is the most common capsular type of meningiococcal disease in the UK?

A. A
B. B
C. C
D. Y
E. W
A

B. B

Because it has only recently been included in routine vaccination.

111
Q

What is the most common cause of death in children under 5 worldwide according to the WHO in 2016?

A

Prematurity

112
Q

Which of the following would be the most appropriate treatment for early onset (<48 hours) neonatal sepsis?

A. I.V. meropenem + aciclovir
B. I.V. cefotaxime + amoxicillin
C. I.V. ceftriaxone + amoxicillin
D. I.V. benzylpenicillin + gentamicin
E. I.V. ceftriaxone + levofloxacin
A

D. I.V. benzylpenicillin + gentamicin

This is the standard treatment for neonatal sepsis, and is chosen because benzylpenicillin treats Group B Strep infection (most common cause), and gentamicin treats E. coli infection (2nd most common cause).

‘B’ is the correct treatment for meningitis in a child under 3 months old - ceftriaxone is avoided because it can cause biliary sludging in these children.

113
Q

VSA:

Which organism is the most common cause of early-onset bacterial meningitis in a neonate?

A

Group B Streptococcus

*Taken from 2017 paper

114
Q

What is the most common cause of late onset sepsis in neonate?

A

Coagulase negative Staph e.g. epidermidis

115
Q

Which specific organism is the most common cause of traveller’s diarrhoea?

A

Entero-toxigenic Escherichia coli

116
Q

VSA:

Name a receptor variation that could be protective against HIV

A

CCR5 mutation

*Taken from 2019 paper

117
Q

VSA:

What is the full name of the organism that causes cat scratch disease?

A

Bartonella henselae

*Taken from 2019 paper

118
Q

EMQ:

Match each disease with the appropriate antiviral treatment. Each option may be used once, multiple times, or not at all.

Oseltamivir
Interferon-alpha
Ribavirin
Foscarnet
Cidofovir
Aciclovir
Ganciclovir
VZIG
Palivizumab
Rituximab
  1. CMV retinitis
  2. HSV meningitis
  3. Varicella-Zoster virus infection
  4. Child with respiratory syncitial virus
  5. Asthmatic with flu
  6. Hepatitis B where the patient wants to try for a cure
  7. RSV prevention in a vulnerable child
  8. CMV infection in a bone marrow transplant patient pre-graft
  9. Must be taken with probenecid
  10. May be taken on its own in patients with resistant CMV
  11. Can be given for post-transplant lymphoproliferative disease
  12. Should be given to a non-immune pregnant woman with chicken pox contact within 10 days
A
  1. CMV retinitis - Ganciclovir
  2. HSV meningitis - Aciclovir
  3. Adult with Varicella-Zoster infection - Aciclovir
  4. Child with respiratory syncitial virus - Ribavirin
  5. Asthmatic with flu - Oseltamivir
  6. Hepatitis B where the patient wants to try for a cure - Interferon-alpha
  7. RSV prevention in a vulnerable child - Palivizumab
  8. CMV infection in a bone marrow transplant patient pre-graft - Foscarnet
  9. Must be taken with probenecid - Cidofovir
  10. May be taken on its own in patients with resistant CMV - Foscarnet
  11. Can be given for post-transplant lymphoproliferative disease - Rituximab
  12. Should be given to a non-immune pregnant woman with chicken pox contact within 10 days - VZIG
    * Modified from 2019 paper
119
Q

Which of the following statements regarding resistance to antiviral drugs is correct?

A. Resistance of HSV to aciclovir is common in the immunocompetent
B. Phenotypic resistance testing is routinely used to detect resistance of CMV to ganciclovir
C. Aciclovir resistance in HSV is most commonly mediated by mutations in the viral thymidine kinase
D. Aciclovir resistance in HSV may also be mediated by mutations in the UL97 protein kinase gene
E. Antiviral drug resistance is most commonly associated with good adherence to treatment

A

C. Aciclovir resistance in HSV is most commonly mediated by mutations in the viral thymidine kinase

95% of cases of aciclovir resistance are due to mutation in the viral thymidine kinase gene (which also confers resistance to ganciclovir), and this almost exclusively arises in immunocompromised patients. The other 5% of cases are due to mutations in viral DNA polymerase.

Mutations in the UL97 protein kinase gene are responsible for resistance of CMV against gancilcovir.

120
Q

Which of the following statements antiviral drugs is correct?

A. Oseltamavir directly inhibits the influenza neuraminidase
B. Zanamivir blocks binding of viral haemagglutinin to host cell sialic acid
C. Oseltamivir inhibits influenza virus uncoating
D. Zanamivir is usually given intravenously
E. Zanamivir is usually given by nebuliser

A

A. Oseltamavir directly inhibits the influenza neuraminidase

Oseltamivir and Zanamivir are both direct neuraminidase inhibitors; Zanamivir is given via a dry powder inhaler or nebuliser, and Oseltamivir (Tamiflu) is given orally. By binding to influenza neuraminidase, these drugs prevent viral escape from infected cells, thereby limiting their ability to further infect the host.

121
Q

A 42 year old woman is admitted with a 2 day history of fever and confusion and presents with new onset seizures.

Which antiviral medication should be commenced asap?

A. Oral aciclovir
B. I.V. foscarnet
C. Oral valaciclovir
D. I.V. ganciclovir
E. I.V. aciclovir
A

E. I.V. aciclovir

Fever, mental state changes, and seizures are highly suggestive of encephalitis, the most common causative organism of which is herpes simplex virus. The treatment for this is a few weeks of I.V. aciclovir.

122
Q

What are the three classes of direct acting antivirals for Hepatitis C?

A

NS5A inhibitors - ‘…asvirs’
Polymerase inhibitors - ‘…buvirs’
Protease inhibitors - ‘…previrs’

123
Q

EMQ:

Match each presentation with the most likely cause. Each option may be used once, multiple times, or not at all.

Mycoplasma pneumoniae
Klebsiella pneumoniae
Streptococcus pneumoniae
Pneumocystis jirovecii
Enterobacteriaceae
Legionella pneumoniae
Mycobacterium tuberculosis
Haemophilus influenzae
Aspergillus fumigatus
Empyema
  1. A previously well 22 year old with cough productive of rusty coloured sputum
  2. A 40 year old man just returned from abroad with a dry cough, malaise, and hyponatraemia
  3. A 45 year old presented to GP with flu symptoms which resolve, 2 weeks later is admitted with bacterial pneumonia
  4. A 62 year old alcoholic man with an upper lobe cavitating pneumonia
  5. A 74 year old who presented with SOB, persistent fever, and pleuritic chest pain and who has failed to respond to broad spectrum antibiotics
  6. A 15 year old with a dry cough, myalgia, fever, and headache. CXR is normal and other kids at school have similar symptoms
  7. A 22 year old neutropenic chemotherapy patient with ongoing fevers unresponsive to antibiotics or antivirals, but with interstitial CT changes
  8. A 70 year old woman admitted for confusion caused by a UTI develops pneumonia whilst in hospital
  9. A 56 year old man presents with cough, haemoptysis and CXR shows a LLL pneumonia. Sputum culture grows Gram- cocco-bacilli
  10. A 21 year old Ecuadorean man presents with a few weeks’ history of cough weight loss, CXR shows RUZ shadowing
  11. A known IVDU who desaturates upon exercise
  12. An 18 year old chemotherapy patient whose CXR shows the ‘Halo’ sign
A
  1. A previously well 22 year old with cough productive of rusty coloured sputum - Streptococcus pneumoniae
  2. A 40 year old man just returned from abroad with a dry cough, malaise, and hyponatraemia - Legionella pneumoniae
  3. A 45 year old presented to GP with flu symptoms which resolve, 2 weeks later is admitted with bacterial pneumonia - Streptococcus pneumoniae
  4. A 62 year old alcoholic man with an upper lobe cavitating pneumonia - Klebsiella pneumoniae
  5. A 74 year old who presented with SOB, fever, and pleuritic chest pain and who has failed to respond to broad spectrum antibiotics - Empyema
  6. A 15 year old with a dry cough, myalgia, fever, and headache. CXR is normal and other kids at school have similar symptoms - Mycoplasma pneumoniae
  7. A 22 year old neutropenic chemotherapy patient with ongoing fevers unresponsive to antibiotics or antivirals, but with interstitial CT changes - Aspergillus fumigatus
  8. A 70 year old woman admitted for confusion caused by a UTI develops pneumonia whilst in hospital - Enterobacteriaceae
  9. A 56 year old man presents with cough, haemoptysis and CXR shows a LLL pneumonia. Sputum culture grows Gram- cocco-bacilli - Haemophilus influenzae
  10. A 21 year old Ecuadorean man presents with a few weeks’ history of cough weight loss, CXR shows RUZ shadowing - Mycobacterium tuberculosis
  11. A known IVDU who desaturates upon exercise - Pneumocystis jirovecii
  12. An 18 year old chemotherapy patient whose CXR shows the ‘Halo’ sign - Aspergillus fumigatus
    * Modified from 2016, 2017, 2018, and 2019 paper i.e. this question comes up, so learn it
124
Q

VSA:

Which organism is the most common cause of bacterial meningitis in a child over 3 months old?

A

Neisseria meningitidis

*Taken from 2017 paper

125
Q

EMQ:

Match each antibiotic with its description. Each option may be used once, multiple times, or not at all.

Teicoplanin
Gentamicin
Amoxicillin
Ceftazidime
Clarithromycin
Piperacillin
Ciprofloxacin
Cefuroxime
Doxycycline

A. Cephalosporin with anti-pseudomonal activity
B. Broad spectrum, no anti-pseudomonal activity, usually given with a beta lactamase inhibitor
C. A glycopeptide used to treat MRSA
D. A protein synthesis inhibitor used to treat Pseudomonal infections but which has no activity against anaerobes
E. Abx of the Macrolide/lincosamide/streptogrammin group used to treat some atypical pneumonias
F. A DNA synthesis inhibitor used to treat Pseudomonal infections but which has little activity against anaerobes

A

A. Cephalosporin with anti-pseudomonal activity - Ceftazidime
B. Broad spectrum, no anti-pseudomonal activity, usually given with a beta lactamase inhibitor - Amoxicillin
C. A glycopeptide used to treat MRSA - Teicoplanin
D. A protein synthesis inhibitor used to treat Pseudomonal infections but which has no activity against anaerobes - Gentamicin
E. Abx of the Macrolide/lincosamide/streptogrammin group used to treat some atypical pneumonias - Clarithromycin
F. A DNA synthesis inhibitor used to treat Pseudomonal infections but which has little activity against anaerobes - Ciprofloxacin

126
Q

VSA:

What does it mean if urine MC&S sample has epithelial cells and mixed bacterial growth?

A

It is a poor sample and was not properly taken mid-stream

*Taken from 2019 paper

127
Q

VSA:

A 34 year old woman returns from 3 months spent in India with fever, constipation, and a relative bradycardia. What antibiotic should she be started on?

A

Ceftriaxone

*Taken from 2019 paper

128
Q

VSA:

An alcoholic man is on ceftriaxone for meningitis, what organism is he at risk of which this does not cover?

A

Listeria monocytogenes

Amoxicillin is added to cover for Listeria

129
Q

Which stain is used to screen for TB?

A. Auramine
B. Sudan Black
C. Congo Red
D. Ziehl-Neelson
E. Perl's Prussian Blue
A

A. Auramine

Auramine staining is faster than staining with ZN (though accuracy is operator dependent) so it is used as a screening test, and Ziehl-Neelson is used for more formal diagnosis.